Estimates of the economic impact of climate change

I realise Stoat has already covered this, but I thought I would also briefly discuss it here. I posted a couple of tweets, that got quite a lot of responses, about the economic impacts of climate change, that tried to address a question that has bothered me (and others, it seems). Essentially, many economic analyses imply that even under quite substantial levels of warming, the economic impact of climate change will be ~10% of GDP and that, because of continued economic growth, this will occur in a world that is substantially richer than we are today. I realise that 10% of GDP is not negligible, but it also doesn’t seem consistent with the suggestion that we’re facing a climate crisis if we don’t soon substantially reduce our emissions. So, what’s going on here? Is it:

  1. The results from these economic analyses do indeed properly reflect the overall impact; the impact could be substantial (~10% of GDP) but we’ll be richer, can more easily deal with these impacts, and really shouldn’t be too concerned.
  2. Even though these economic analyses are reasonable, they’re really order of magnitude estimates, and so we should interpret ~10% as representing a large impact.
  3. Globally, these economic analyses are reasonable, but they largely ignore the distribution of the impacts; losses could be huge in regions that are poor and don’t contribute much to global GDP.
  4. The results from these economic analyses do indeed properly reflect the economic impact, but miss other substantive impacts that can’t easily be quantified.
  5. These results don’t really make sense if we end up warming by more than 3-4oC.

I should make clear that I’ve updated my points to include some suggested by other (H/T Andrew Dessler).

I don’t have a particularly good answer to my posed question. If pressed, I would go for a combination of 2, 3, and 4. It seems clear that there are large uncertainties, they clearly ignore some impacts that are difficult to quantify (the loss of coral reefs, for example), and probably under-estimate – or appear to – the human impact in regions that are poor. However, I still have a problem reconciling these type of estimates with the sense of urgency coming from others.

Maybe one solution is to give more weight to moral arguments (H/T Kyle Armour):

By what natural authority do we accrue the right to materially reengineer the only planet in the vast immensities of the universe known to harbor life?

On the other hand, maybe these economic analyses are still useful for guiding how we respond to climate change. For example, even though some of these analyses suggest that the impact could be modest, they do still suggest that there is a net benefit to emission reductions.

What I would really like is to better understand the conditions under which these economic analyses are regarded as reasonable. It seems clear that there are levels of warming beyond which the damage estimates are clearly nonsense (DICE, for example, suggests that 10oC of warming would produce damages of about 28% of GDP). However, for relatively low levels of warming, maybe they’re fine. Where is the actual boundary? Also, maybe it needs to be clearer as to the goals of such modelling. Is it really to try and determine the economic impact in 2100, or is it mostly to guide what we do in the next decade, or so?

Something that I posed during the Twitter discussion was whether or not one could sanity check these kind of estimates. In physics, one can often do a basic back-of-the-envelope type of calculation to check if the result from a more complex analysis actually makes sense. Is that possible in this context, or is it simply too complicated? If the latter, what does this imply? Should we simply trust the numbers that pop out of these calculations, or can we say “sorry, that doesn’t seem to make sense”? If the latter, how does one avoid motivated reasoning?

Again, I don’t have good answers to these questions, so I’ll stop there. I’ll post some links below, some of which are papers suggested by Genevieve Guenther, that I really should find time to try and read.

Links:
Costing the Earth: A Numbers Game or a Moral Imperative? (paper by Gerard Roe suggesting that we should give greater weight to moral arguments).
Long-Term Macroeconomic Effects of Climate Change: A Cross-Country Analysis (paper suggesting that the economic impact of ~4.2MoC of warming would be ~7% of GDP, but which does suggest that this is probably conservative because it ignores rare disasters).
On Modeling and Interpreting the Economics of Catastrophic Climate Change (paper by Martin Weitzman about fat tails).
Limitations of integrated assessment models of climate change (paper by Frank Ackerman suggesting that IAMs have some serious limitations).
Climate Change Policy: What Do the Models Tell Us? (paper by Robert Pindyck suggesting that IAMs have fundamental flaws).

This entry was posted in Carbon tax, Climate change, economics, Global warming, Philosophy for Bloggers and tagged , , , , , , , . Bookmark the permalink.

200 Responses to Estimates of the economic impact of climate change

  1. Biff Vernon says:

    I’d add to the reading list the recent paper by Thompson and Smith in Economics Vol. 13, 2019-40 | October 08, 2019 | http://dx.doi.org/10.5018/economics-ejournal.ja.2019-40

  2. Jai Mitchell says:

    Sumply assume an historic growth pattern and project it to 2100. Assume unlimited physical resources. Develop a spacial map projecting changes in habitable/cultivatable lands and superimpose the 2100 society onto the new grid. Disregard adaption costs and potential societal impacts (i.e. conflict) and. . . Voila!

  3. Jai Mitchell says:

    Ackerman and Stanton did a comprehensive takedown of the IAMs in 2012. They looked at how cross contamination of one models assumptions would behave on other models and took the median social cost of carbon value. Def worth a read. http://www.economics-ejournal.org/economics/journalarticles/2012-10

  4. Hank,
    Thanks, I wasn’t aware of that site.

  5. lerpo says:

    3. Globally, these economic analyses are reasonable, but they largely ignore the distribution of the impacts; losses could be huge in regions that are poor and don’t contribute much to global GDP.

    Even within rich countries, the distribution of the impacts may mean that the cost as a percent of GDP is minimal, but the impact to those affected is catastrophic.

    For example, “Morgan Stanley says climate-related disasters like hurricanes and wildfires have cost North America $415 billion, or two-thirds of the global total.” But that only amounts to “0.66 percent of North American GDP.”.

    0.66 isn’t much but anyone who lost their neighbourhoods or families in one of those wildfires would probably not see the impacts as trivial.

  6. lerpo,
    Indeed, the impacts can be widely distributed within countries as well as globally.

  7. Biff said:

    “I’d add to the reading list the recent paper by Thompson and Smith in Economics Vol. 13, 2019-40 | October 08, 2019”

    Now that the concept of the Hawkmoth effect has gestated a bit, it appears to come from long-held concepts in structural stability.

    https://en.wikipedia.org/wiki/Structural_stability“Unlike Lyapunov stability, which considers perturbations of initial conditions for a fixed system, structural stability deals with perturbations of the system itself.”

    Not very difficult to find systems that are touchy to their parameters, and it is of course frustrating to deal with them. I think it’s one reason that the engineering concept of feedback is often used, as negative feedback will transform a system that is sensitive to parameter selection (such as an open-loop amplifier) to something that is structurally stable.

    the way I look at it is that geophysics-based behaviors on their own can’t come up with mechanisms of smart feedback and either end up in stable end-point states or endlessly oscillate around or react to external forcings

  8. …and Then There’s Physics says:

    Biff,
    Thanks. I have been meaning to read that. I haven’t entirely agreed with some of what they’ve suggested about models.

    For me, it’s not that I don’t agree with what they say, but that there isn’t much one can do about it. Whenever there are slight differences in material characteristics that can result in huge differences in behaviors, it will be much more challenging to make a model useful for predictions. That doesn’t mean that we give up, but just have to work that much harder to model things that we have no control over.

  9. Something that I posed during the Twitter discussion was whether or not one could sanity check these kind of estimates. In physics, one can often do a basic back-of-the-envelope type of calculation to check if the result from a more complex analysis actually makes sense. Is that possible in this context, or is it simply too complicated? If the latter, what does this imply?…

    Essentially, it is not possible in this context – at least in 2019… As Weitzman and Wagner say (below): “We just don’t know. And no one else does, either.”

    But what that deep uncertainty implies is that there’s a cost to this uncertainty itself, and we “should” be hedging against erring to the low side of mitigation, in case we are wrong.

    The sensitivity of the damages to something as basic as the functional form of the damage function, and really having no idea whether it is quadratic? exponential? other? – should really be driving home the need to invest contingently on the chance that we might be far too optimistic in the consensus economic cost projections. Although it seems we are interpreting things the exact opposite way.

  10. My list of reasons that IAMs may not fully represent climate damages:
    1) undervalued impacts (e.g., poor people, existence value of ecosystem diversity)
    2) impacts that aren’t included in the IAMs (e.g., climate -> air quality -> mortality link)
    3) exclusion of climate impacts on _growth_ in addition to single-year GDP
    4) assumption that we can maintain 2% growth to the end of the century
    5) excess optimism about adaptation (whether to heat waves or coastal flooding or agriculture)
    6) the possibility that models are missing potential highly non-linear changes that may occur at temperatures that are unprecedented in the past million years

    Having said that: I also don’t see a high likelihood* climate change seriously interfering with the ability of the developed world to keep an industrial society running… so if we don’t care about diverse ecosystems, or about the welfare of the most vulnerable, then with just a bit more allocation of resources towards air conditioning and coastal protection and shifting agricultural lands, the bulk of the industrialized world will probably do okay.

    *But there is a non-zero chance that there could be more fundamental disruptions than I anticipate. Which is hard to value. I don’t agree with the Weitzman “fat tail” for climate sensitivity, but I think that there is still a possible “fat tail” for damages…

  11. climatemusings,

    But there is a non-zero chance that there could be more fundamental disruptions than I anticipate. Which is hard to value. I don’t agree with the Weitzman “fat tail” for climate sensitivity, but I think that there is still a possible “fat tail” for damages…

    Interesting. I had thought something similar. As I think James Annan points out, the possible fat tail in climate sensitivity is more because actually estimating the single value that it has is difficult, rather than it being an indication that there is some reasonable chance of it actually being very high. However, it seems distinctly possible that society may respond to climate change in very unexpected ways.

  12. In that when I link pics/text via twitter, it seems pick up the preceding one as well, feel free to eliminate my first link above (the one with the entirely unrelated plots from Myles Allen’s paper). And then maybe kill this comment too.

  13. Ben McMillan says:

    Things like DICE are pretty hopeless as long as there is a big change. And 10% is actually a big change. You could set agricultural output to zero, and burn back all the forests to bare rock and DICE will give a few percent reduction in economic output. Population doesn’t change (since this is ‘exogenous’)

    Something different happens in reality.

    People in DICE would also happily exterminate every other living creature that doesn’t play an industrial role in order to gain a couple of percent of their salary. This, maybe, I can believe.

  14. Joshua says:

    Anders –

    Was gonna make a comment about limits of GDP as a metric, but see that lerpo beat me to it. But so as not to waste a click-through…

    > Even though these economic analyses are reasonable, they’re really order of magnitude estimates, and so we should interpret ~10% as representing a large impact.

    I can’t quite figure out what that means. Could you explain? Are you just saying that if GDP growth is larger then the impact will be larger?

  15. Joshua,
    It was Andrew Dessler who made that point, initially. However, what it means is that these are so uncertain that we should probably regard an estimate of ~10% as being 10s of percent, rather than precisely 10%.

  16. BBD says:

    I’d incline towards the concerns of agronomists, ecologists and military planners over economic models of questionable utility when it comes to assessing future climate impacts.

  17. WMC,
    I’m not that surprised 🙂

  18. It really seems to miss the mark to be concerned about the economic costs of triggering an extinction event, don’t you think? I don’t know if I think of it as a moral argument, but the indigenous thought about planning for seven generations is seriously conservative in the best sense of that word. Talk with Greta about the economic impact of global warming on her grandchildren and see how you do. I suspect you would get an earful of passion and uncommon sense.

    so, yeah… the economics of global warming are going to be a real drag.

    Cheers,

    Mike

  19. Michael 2 says:

    At least in the United States, the relentless growth of national debt is a chicken growing very large and is going to come home to roost a lot sooner than sea level rise eats New York City.

  20. -1=e^iπ says:

    6. The economic analyses are reasonable, climate change is a problem and the optimal solution is a global pigouvian tax. However, the premise that there is a climate crisis is incorrect. Climate crisis is not an accurate description of reality; it is a political claim popularized by the guardian newspaper earlier this year.

  21. Willard says:

    Did someone mention agriculture?

    Everything is fine in that world:

    What does it have to do with climate change, you may ask? This:

    The public is providing more than $1m per minute in global farm subsidies, much of which is driving the climate crisis and destruction of wildlife, according to a new report.

    Just 1% of the $700bn (£560bn) a year given to farmers is used to benefit the environment, the analysis found. Much of the total instead promotes high-emission cattle production, forest destruction and pollution from the overuse of fertiliser.

    https://www.theguardian.com/environment/2019/sep/16/1m-a-minute-the-farming-subsidies-destroying-the-world

  22. angech says:

    ATTP
    “Essentially, many economic analyses imply that even under quite substantial levels of warming, the economic impact of climate change will be ~10% of GDP and that, because of continued economic growth, this will occur in a world that is substantially richer than we are today.”
    A point you might find very relevant to your concerns is that the more “successful” and productive the economy is in a non adapting world the more GHG and GHG effects and damages you would have to postulate.
    A sluggish economy or depression might actually reduce the risk compared to a gangbusters 10 year economy. A substantially richer world might cause far more than 10% damage.

  23. David B. Benson says:

    Academic view of the effects of agriculture:
    https://m.phys.org/news/2019-10-zeroing-food-agricultural-impacts-climate.html

    Not good.

  24. Steven Mosher says:

    “Did someone mention agriculture?”

    Food like college and health care should be free. We need to increase subsidies to farmers.

  25. Steven Mosher says:

    “The results from these economic analyses do indeed properly reflect the economic impact, but miss other substantive impacts that can’t easily be quantified.”

    Yes, if we could put other things in the model the results would be different This is like a judith argument about missing processes in GCMS.

  26. David B. Benson says:

    Something is killing the insects:
    https://m.phys.org/news/2019-10-insect-decline-extensive.html

    I think this is global in scope.

  27. -1,
    That’s just another statement. I’m looking for an explanation as to why that is a correct statement. (Also your 6 is essentially my 1).

  28. izen says:

    The claim that with future Grooowth of the wealth of Nations will enable them to pay for, or offset, the costs of climate change has always struck me as ridiculous.

    If Bangladesh gets ten times as rich as it is now, its GDP/capita increases by an order of magnitude, then is becomes about as ‘rich’ as Mexico.
    The idea that a Nation like Mexico is wealthy enough to weather the sort of impacts of climate change that Bangladesh is likely to face….
    I think this fails the sanity check.

  29. David B. Benson says:

    I agree with izen. So far this exercise is only fit for the common room in Bedlam.

  30. Ben McMillan says:

    Even the people who write IAMs are quite happy to point out that they are much less mature and well founded, and have huge error bars compared to GCMs. GCMs are actually largely based on applying well-tested fundamental physics concepts, and have been carefully validated and verified (as have related models like weather codes). IAMs, not so much.

    So it isn’t exactly heresy to suggest that we shouldn’t take the results of IAMs as some kind of ‘gold standard’. Apart from anything else, they are structurally incapable of modelling large perturbations, which is exactly the thing that we need to worry most about (ie, point 5).

    That is, the understanding from ecologists, historians, and so on should be taken to have roughly equal standing to things like IAMs. Even for climate sensitivity analysis, GCMs are only one tool in the kit…

  31. Greg Robie says:

    If one feels motivated reasoning is avoidable, doesn’t one need to think different?!? 😉 The species’ predilection toward motivated reasoning is [very likely] endemic. 😉 Because of this I posit that our social as-good-as-it-gets is a consciousness of the motivated reasoning that is already socially embedded in trusted homeostases.

    Therefore, might it be helpful/insightful to try to imagine what an amalgam of 2, 3, and 4 adds up to in terms of motivated reasoning?

    A view of human history I’ve found insightful is a framing that defines its cycles in terms of the accumulation and redistribution of “wealth”. The choice our species may have agency around is whether the redistribution is socially experienced as violent or non-violent. CapitalismFail’s Anthropocene’s klimakatastrophe is the externalized violence of the trusted economic meme coming home to roost. Regardless of our plausible agency and the emergent properties of complex systems, chaos cometh.

    Thomas Jefferson is credited with saying that constant vigilance is the price of freedom. Systemically that is both cogent and prescient. Thanks to limited liability laws and the ‘free’ markets they engendered, both “wealth” and “freedom” are currently felt to be the right to be irresponsible. What could go wrong?!? 😉

    The functional nature of motivated reasoning among a socially privileged is that it blinds. Fundamentally, all five options are delusional. But for computers storing, and the digital backup of those measurements stored in places like Iron Mountain – just north of me, they do not exist (except within a trusted motivated reasoning/social convention/social contract. If this assertion challenges the imagination, imagine digging up J. P. Morgan’s corpse and seeing how much wealth he took with him. By social conventions his offspring retain an iteration of it, and (based on conversations I had with fourth generation removed decendants I had at a third generation’s decedent’s funeral this fall), retain the delusion … & passionately so. It is this passion that tends to trigger motivated reasoning. And one does not need to be a Morgan descendent to be addicted to it!

    …& then there is the physics of the latent heat of Arctic sea ice. If the NSIDC 1981 – 2010 average extent loss of 9.02 million square kilometers is equivalent to the 5 x 10^21 joules that PIOMAS calculates as the Arctic’s melt season total, this year’s 10.08 million square kilometer loss is an 11% increase … and about a 6 1/3 year doubling rate. Or, the 8.6 x 10^19 J/yr of ongoing and committed warming may be 9.5 x 10^19 J/yr this year.

    But hey, ~90% of that was absorbed … so there is no committed [short term] warming regarding averaged global surface air temperatures after 6 – 30 years (with the cryosphere dynamics excluded)!

    Or, another example of trusted/hidden in plain sight motivated reasoning?

    =)

    sNAILmALEnotHAIL …but pace’n myself

    https://m.youtube.com/channel/UCeDkezgoyyZAlN7nW1tlfeA

    life is for learning so all my failures must mean that I’m wicked smart

    >

  32. Chubbs says:

    Its hard to get too excited about economic model forecasts for 2100 without a proven track record of accurate prediction. One thing is becoming clear, fossil fuels are starting to loose competitiveness due to resource limitations and reduced costs for alternatives. Guessing that rapid growth won’t persist to 2100 unless fossil fuels are supplanted.

  33. afeman says:

    https://statmodeling.stat.columbia.edu/2019/11/01/the-environmental-economics-echo-chamber-gremlins-and-the-people-including-a-nobel-prize-winner-who-support-them/


    In the 2013 version of DICE, Nordhaus based his monetary climate-damage function on Tol’s 2009 JEP results. That function connects any amount of warming to the resulting loss in GDP. Here are Nordhaus and Storzk, p. 11 of the 2013 DICE user’s manual: “DICE-2013R uses estimates of monetized damages from the Tol (2009) survey as the starting point. . . . I [sic] have added an adjustment of 25 percent of the monetized damages to reflect these non-monetized impacts. While this is consistent with the estimates from other studies (see Hope 2011, Anthoff and Tol 2010, and FUND 2013), it is recognized that this is largely a judgmental adjustment.” Notice that he uses FUND, Tol’s IAM, as a benchmark. My function looks like Tol’s, he seems to be saying, so I’m good. This is the echo chamber.

  34. Farm subsidies, like fossil fuel subsidies, aren’t “given” to farmers. Their purpose is to make food, and energy, cheaper in cities and suburbs. Because that’s where the people are. Take them away and the only impact is that food and energy become more expensive in Brooklyn and Chicago.
    You can use subsidy policy to drive behavior, but for political buy-in it is better to be direct about what you’re doing. Reducing beef subsidies = steak and hamburger cost more in NYC. You want people to be happy with the trade-off, not surprised and angry about it.
    It sounds like a simple point, but it’s a serious cause of inaction. There is no shortage of politicians willing to say they wouldn’t dream of raising the gasoline tax in the US (because that would hurt you, soccer mom!) they’re only going to take away fossil fuel subsidies which only impacts greedy corporations!
    Because they know the latter has the same impact on the price of a gallon of gas (or more), those politicians actually do neither. Then blame money in politics. All because they sold a unicorn rather than attempted to get buy-in on what they want.

  35. afeman,
    Thanks, I’d completely missed those recent posts by Andrew Gelman. Quite something.

  36. afeman says:

    He’s definitely worth following, though most of what he posts isn’t as relevant to climate as this.

    Somehow I’m reminded of how, at Keith Kloor’s old blog, Judith Curry once suggested that the IPCC process had been corrupted, then walked it back by claiming she was talking about data corruption.

  37. wmconnolley says:

    > posts by Andrew Gelman. Quite something

    I don’t think it gets you very far, if anywhere. You’ve still got nothing to put in place of the DICE damage function, or any real evidence that it’s wrong (evidence that it isn’t solidly based isn’t evidence that it’s actually wrong). If things were really all so terrible, why isn’t some eager young bunny making their name producing something better?

  38. wmc,

    If things were really all so terrible, why isn’t some eager young bunny making their name producing something better?

    Indeed, this is a fair point. My suspicion is that this is a very trick problem. Whatever you do, you probably have to trust that your model works out of sample. I read some of Martin Weitzman’s paper last night. It pointed out that the damage function could have been chosen to be something like \exp[\gamma (\Delta T)^2], rather than going something like \gamma (\Delta T)^2 and the damages would be very similar for small \Delta T, but very different for large \Delta T. (I may have missed some signs and constants). There probably is no robust way to determine which functional form is most reasonable.

  39. Joshua says:

    I will just always have a problem with the X% of GDP argument related to outcomes of climate change – except in a particular context:

    So if there really are increased extreme events due to climate change, or sea level rise really does require group migration, would we just say to people something on the order of “Well, I feel bad for you but the damages you suffered amount to only 10% of global GDP?” At some point, at least in theory, there will be some differential increase in the number of people who suffer some significant form of damage as the result of climate change. Translating that differential increase in damage to a (supposedly relatively small) decrease in GDP growth doesn’t make a lot of sense to me – except as a reason to not do something to mitigate that differential increase in damages (or, legitimately, to engage in the discussion over whether we’re facing a “crisis”).

    Now an argument is often made that mitigating that differential increase in damages will actually cause a differentially greater decrease in economic growth – and at that point then I think there is some reason to make that translation. If what you’re arguing is that mitigation will translate to more suffering than the increase in suffering attributable to climate change, then that seems like a discussion worth having.

    But don’t hand wave towards that discussion with vague generalities and then tell me that it’s just too hard to measure (positive and negative) externalities. Don’t piss on my leg and tell me it’s raining.

  40. Ben McMillan says:

    Pindyck’s “Use and Misuse of Models” paper gives you some options once you admit that the DICE damage function (among other things) is totally dodgy (sorry, is not ‘well founded’).

    Basically
    1) Listen to people who aren’t economists.
    2) Calculate the cost of carbon to be whatever is necessary to stay below 3C.

    Predicting the coupled economy-social-ecological history of the next 100 years is a stupendously hard problem. So you can do simple and clearly wrong (DICE) or less simple and still wrong. Basically working in the field of IAMs requires either a vast degree of hubris or a willingness to admit you are working with a caricature of the real world.

  41. Willard says:

    > You’ve still got nothing to put in place of the DICE damage function

    Our Stoatness’ squirrel is duly noted. Let’s start with the beginning of the post:

    Jay Coggins, a professor of applied economics at the university of Minnesota, writes in with some thoughts about serious problems of within the field of environmental economics:

    Your latest on Tol [a discussion of a really bad paper he published in The Review of Environmental Economics and Policy, “the official journal of the Association of Environmental and Resource Economists and the European Association of Environmental and Resource Economists.”] got me [Coggins] thinking. People might suppose the gremlins hubbub dinged his reputation, but no. Tol is still considered an elite climate economist. In 2016, when a lineup of heavy climate-econ hitters wrote a Policy Forum piece for Science, Tol was included as a co-author. That paper is safely post-gremlins; his reputation remains intact.

    https://statmodeling.stat.columbia.edu/2019/11/01/the-environmental-economics-echo-chamber-gremlins-and-the-people-including-a-nobel-prize-winner-who-support-them/

  42. Willard says:

    > If things were really all so terrible, why isn’t some eager young bunny making their name producing something better?

    https://twitter.com/nevaudit/status/1186771743534395393

  43. Greg Robie says:

    Corrections: FWIW, I do this commenting from my deceased father-in-law’s 4s iPhone … that has a dead region at the bottom of the screen. That, fat fingers, and motivated reasoning conspired this morning to make me not see the intended 9.01 was entered as 9.02, and 10.09 was off by a digit … should such matter.

    But reviewing the PIOMAS data and some previous assertions, as part of composing an email to Susan Solomon (after watching the YouTube of her keynote that @ecoquant shared – & this is a more direct link: https://climate.mit.edu/video/mit-climate-action-symposium-progress-climate-science-part-2), I find that motivated reasoning has blinded me a couple of times.

    My attempts to roughly quantify the un-modeled forcing that the Inuit’s observations of increased refraction point to have involved numbers slightly larger than what the 25′ tape measure my blue collar day job utilized (& thereby informs my motivated reasoning). It turns out I’ve been conservative in the figures I’ve shared. In terms of the measurement of joules I presented in the Black Knight comment thread, this was by 170%. Then, and more recently, I read the PIOMAS’ 5 x 10^21 J/yr as that amount of current warming that will be warming something else when an annual blue ocean event warming threshold is crossed. It looks like that amount of heat is – calculated proportionately – 8.56 x 10^21 J/yr. (but I think that amount of warming is also during the Arctic’s melt season. What is becoming available to warm something else regarding the diminishing sea ice’s latent heat of ice as a heat storage function will max out at that number. In the polar night radiant cooling will be ongoing after the annual blue ocean threshold is crossed. Without an insulating cover of ice, perhaps such radiant heat loss will be a greater loss of heat than is represented in the 5 x 10^21 J that PIOMAS calculations currently identify as being stored during the night to reduce the amount of committed heat that an echo chamber’s motivated reasoning is discounting during the polar day. And/or, maybe the annual Arctic blue ocean threshold will be integral to the loss of the Arctic’s (and/or the planet’s) cryosphere (with sheet ice batting cleanup)?

    Regardless, and if motivated reasoning hasn’t got the better of me again, there are bigger numbers than what I’ve stated that are associated with the consequence of the industrial revolution’s fossil carbon soot perturbing the Arctic’s upper troposphere in its unique twilight within its polar day’s dynamics over the past couple of centuries. So, given motivated reasoning’s social role, are any numbers concerning an unconsidered regional and seasonal forcing more than what existing scientific conventions/motivated reasoning (& the need to accommodate historic and current computer power limits) can readily countenance?

    @-wmconnolley, isn’t the “chaos cometh” thing I’ve argued proven by this surety – and rephrased and applied differently: evidence that G_d does not exist that isn’t solidly based on the existence of G_d isn’t evidence that G_d doesn’t exist?

    …And the reason “eager young bunnies” are missing in action is that they see, and are waking up from, the motivated reasoning that has elicited close to 200 years of deep privileged social slumber … [highly likely] it has been longer and/or deeper within the confines of the walls of the ivory towers?!? 😉

    sNAILmALEnotHAIL …but pace’n myself

    https://m.youtube.com/channel/UCeDkezgoyyZAlN7nW1tlfeA

    life is for learning so all my failures must mean that I’m wicked smart

    >

  44. izen says:

    @-wmc
    “If things were really all so terrible, why isn’t some eager young bunny making their name producing something better?”

    Because the field is defined and ruled by strict adherence to mathematical formalism.
    Accurately reflecting the complex multi-factorial interactions that shape econometrics is abandoned in favour of internal consistency of the mathematical modelling.

    A model that avoids significant discontinuities and predicts a calculable reduction in GDP as a result in food prices as acceptable.
    A model that predicts a civil war, mass movement of refugees, the involvement of super-powers adopting that war as a proxy battlefield. The opportunistic expansionism of neighbours into that war and the rise of radical ideologies driving a global terrorist movement would not get accepted into the literature…

  45. izen says:

    That should be –
    ‘…calculable reduction in GDP as a result OF DROUGHT AND A RISE in food prices as acceptable.
    (Syria is the obvious reference)

  46. wmconnolley says:

    I didn’t realise you were so familiar with the econ academia. I’m certainly not.

    You can have the same answer we give the denialists, when they complain they and their brilliant friends are censored in the GW literature: it isn’t true that brilliant new results can’t get published. That academia is full of bright young things keen to make their name by overturning accepted wisdom.

  47. Joshua says:

    > I don’t think it gets you very far, if anywhere.

    It gets you pretty far w/r/t evaluating how much trust, people who can’t evaluate the economics of climate change on their own, should place in the conclusions drawn by Tol, and the conclusions of people who rely on Tol – and by extension Nordhaus’ work that relies on Tol’s work.

    Gelman’s larger focus is on problems within published scientific research more generally, not so much on settling issues of climate/economics research.

    It is interesting that some of the people who get the vapors so often about problems in climate science research (including Tol) don’t seem particularly *concerned* about problems in Tol’s research related to climate change.

    I wonder why that is.

  48. Willard says:

    > I didn’t realise you were so familiar with the econ academia.

    Second time our Stoatness dodges the fact that the criticism comes from academic economists.

  49. izen says:

    @-wmc
    “I didn’t realise you were so familiar with the econ academia. I’m certainly not.”

    Nor am I.
    But I have read a certain amount of the critiques within the field. I would regard it as a similar error to the claims of biological determinism when approaching social/ethical issues. So just on the basis of –
    ‘Know the enemy…’

    https://uhra.herts.ac.uk/handle/2299/5439
    “Economists have converted the subject into a sort of social mathematics in which analytical rigour is everything and practical relevance is nothing. I believe that on this issue, Lawson, Blaug and I are in agreement: the victory of technique over substance is a chronic problem within modern economics.”

    See also

    https://economistsview.typepad.com/economistsview/2009/03/mathematical-formalism-in-economics.html

    https://riu.austral.edu.ar/bitstream/handle/123456789/342/71%20crespo2016.pdf?sequence=1

    Click to access Dimitris_Milonakis.pdf

    “Excessive mathematisation and formalisation of economic science has been one of the most important features of the development of economic science in the later part of the twentieth century. What were the causes behind this excessive mathematisation of economic science? Why did it happen when it did?”

  50. bjchip says:

    The mainstream of economics is enamored of the rolling of DICE. The problem is that its projections of damage neglect every social change required to adapt and neglect all the probable non-linearities.

    Here’s an economist who “gets it” and throws it back in the faces of people like Tol and Nordhaus.
    https://evonomics.com/steve-keen-nordhaus-climate-change-economics/

  51. bjchip says:

    “What were the causes behind this excessive mathematisation of economic science? Why did it happen when it did?””

    I make the answer to this “Physics Envy”.

    Economists never defined money and do not actually know what it is, only what it does under certain conditions. As a result each measures economic success with a different rubbery meterstick.

    Real Money represents work done. That’s work as defined by physics. The laws of thermodynamics actually apply, which means entropy has to apply to money (as demurrage) and money cannot make more money and ownership cannot make money.

    This is not a simple thing to correct having gone so far down the path of money represents debt (as it has for the past 1000 years and has exclusively done for the past 50.

  52. bjchip says:

    I see now what Stoat has to say about Keen, but he (Stoat) leaves out the problem of air conditioning for the ordinary folks in those countries when he dismisses the problem of major cities needing to be moved. Air conditioning for the people who work in the fields and on the docks and who have a tendency to civil unrest when they are neglected and hungry. The cities are not small and are already borderline. The non-CO2 emitting power supply is not being built.

    Stoat also neglects the actual statistical treatment of the tipping points. Which makes up the bulk of Keen’s commentary about the “Damage Function” that Nordhaus & Tol use.

    So the very reference that Nordhaus uses to justify not having a tipping point in his Damage Function establishes that his Damage Function should have a tipping point.

    Keen provides a revised damage function.

    “This can easily be illustrated by replacing Nordhaus’s quadratic with a very similar one that does have tipping points: a “rational function“. This is a fraction with one polynomial divided by another. Since Nordhaus’s function is just a square of the (difference in) temperature, the required function has a constant times temperature to the third power on the numerator, and a linear function of temperature on the denominator.”

    and uses it.

  53. BBD says:

    “Economists have converted the subject into a sort of social mathematics in which analytical rigour is everything and practical relevance is nothing. I believe that on this issue, Lawson, Blaug and I are in agreement: the victory of technique over substance is a chronic problem within modern economics.”

    It happened in jazz, too, and eventually gave us Math Metal, so the antecedent is not encouraging.

  54. Arthur Smith says:

    I think the main problem here is GDP doesn’t measure everything we care about. What would economic models say is the GDP impact of the deaths of 1 billion people? Probably less than 10% – maybe far less if those killed were the poorest among us. But if we knew a billion lives were at stake, the world would surely put in enormous efforts to prevent it – 100s of trillions of dollars worth perhaps if there was sufficient time to gather those resources.

  55. Willard says:

    > It happened in jazz, too, and eventually gave us Math Metal, so the antecedent is not encouraging.

    Bach would resemble that remark.

    In other news:

  56. izen says:

    @-bjchip
    “I make the answer to this “Physics Envy”.”

    I am dubious that this is a complete answer.
    Since Newton attempts have been made to use the mathematical tools derived from Physics to provide explanations of economics. But until the 1930s-1950s these were overshadowed and got minimal attention compared to the rhetorical arguments based on chains of conceptual reasoning.
    This link I gave earlier –

    Click to access 71%20crespo2016.pdf

    provides an interesting historical analysis of the emergence of strict mathematical formalism in economics, rooting it in the changing philosophical attitudes within mathematics and the rise of absolutists concepts of the TRUTH function in formal systems advanced by Hilbert, Russell and Moore.
    A program subverted by Godel and Church-Turing.

    I suspect there were also political and ideological motives for a desire to ground economics in a discipline regarded as a source of infallibility and absolute consistency after the crash of the 1930s and in response to the alternative economics of communism during the cold war.
    How politically convenient to have a system of economics that could be regarded as absolute Truth, which also delegitimised your political adversaries.

    @-“Real Money represents work done. That’s work as defined by physics. The laws of thermodynamics actually apply, which means entropy has to apply to money…”

    Again I would have reservations.
    There is a seductive claim that money is a ration token for scarcity, and that rather than a Gold Standard, it could be backed by being a representation of available Joules.

    But as in Thermodynamics, in an open system rather more complex and iterative processes may be important. The flow of energy, or money, through a system can engender increasing levels of order and pattern which have important effects beyond those measured by a simple change in entropy (or debt).
    Not least the evolution of biological systems and most recently, sentient intentionality from which emerges the self-referencing circularity that invalidates ‘rational agent’ hypothesises and bedevils any attempt at finding stable equalibria, or even consistent patterns of behaviour.

  57. -1=e^iπ says:

    ATTP –

    “That’s just another statement. I’m looking for an explanation as to why that is a correct statement. (Also your 6 is essentially my 1).”

    Why does it seem intuitive to you that warming of the planet by a few degrees would only have a relatively minor impact on GDP. Does the temperature or rainfall outside affect one’s productivity if they are working on a computer inside an climate controlled office? If I move from Ottawa, Canada to Brisbane, Australia, would my productivity decrease by 90% due to the change in the average local temperature?

    If anything, two of the main economic IAMs (DICE and PAGE) overestimate the impact of climate change on GDP because they neglect that as countries get richer, their GDP depends less strongly on the local climate. FUND does take this into account, which is one of the main reasons it has lower social cost of carbon estimates.

    On the unrelated topic of Weitzman, [Snip. Not funny. -W]

  58. Greg Robie says:

    @- bjchips, “physics envy” – brilliant! =)

    And this is the second time Keen has been advanced here recently as an argument against DICE. As we saw with @wmc’s previous example of motivated reasoning, rational criticism is not very relevant.

    While Dame Nature does hold the trump card regarding the collapse of globalized CapitalismFail, the actual social tipping will likely not take that long.

    MIT’s John Reily at 27:00 – 32:17 here – https://climate.mit.edu/video/mit-climate-action-symposium-progress-climate-science-part-4 – the first MIT climate symposium @ecoquant shared links to, concludes similarly re Nobel Nordhaus and Marty Weitzman’s “Dismal Theorem”. Recall Carter’s malaise speech and a pattern become plausible. On those rare occasions when truth is spoken publicly, spin doctors flood the air waves.

    Given the timing of Nordhaus’s shared Nobel and the constraints on closing the first meeting of the parties to the Paris Agreement (CMA1), particularly because of the relevance of such and the role of science going forward, the Noble was, from this perspective, brilliantly executed game theory strategy. “Fake” economic models clothed in Nobel raiment is effective gaslighting regarding – in terms physics defines – meaningless aspirations.

    But back to the timing regarding chaos and predictive models. Call it P-envy or hubris, it happens. sNAILmALEnotHAIL …but pace’n myself

    https://m.youtube.com/channel/UCeDkezgoyyZAlN7nW1tlfeA

    life is for learning so all my failures must mean that I’m wicked smart

    >

  59. Greg Robie says:

    Touch screen pre-fire. More [maybe] later. =)

    sNAILmALEnotHAIL …but pace’n myself

    https://m.youtube.com/channel/UCeDkezgoyyZAlN7nW1tlfeA

    life is for learning so all my failures must mean that I’m wicked smart

    >

  60. David B. Benson says:

    Exploring the origins of mathematics in economics
    https://m.scirp.org/papers/63422

    4th century BCE.

  61. Greg Robie says:

    @-izen. Interesting speculation regarding mathematical models. Isn’t it just as probable that the development of computers caused the shift? Regardless, doesn’t the preponderance of the human species run on stories? Few of us are left brain enough to even have an option of doing otherwise! 😉

    So a story as an anecdote. My father was about 25 when he flew the last US Army Air Corp transport out of China as the Nationalist government collapsed. Student agitators were integral to that collapse. Fast forward to the spring of 1970. The student strikes culminating in Jackson State and Kent State truncated that academic year. The conversations I brought home to my parents dinner table conflated Vietnam protests with the society destroying student unrest my dad had witnessed. When they had dropped me off at college the previous fall the information tabling the SDS was doing had apparently caught my dad’s attention. His gut reaction to my questions about the status quo was irrational. He said he knew he should have withdrawn me from college that first day. He feared student led chaos. Another fast forward, this time about three years and the TV is showing the evacuation of the American Embassy in Saigon. Helicopters landing filling and departing from a rooftop helipad. Subsequent footage show those same helicopters being pushed off to stern of an aircraft carrier. Chaos left behind. If those dynamics involving chaos can be modeled mathematically, they sure can’t be summed up and communicated with numbers for and to the majority of us … and, in emotionally significant situations, majorities have a way of acting irrationally, thanks to motivated reasoning!

    A speculative anecdote regarding MIT. About five years ago a group of student activist were pushing for MIT to divestment from fossil carbon companies. Twitter provide a window into this effort. It failed. The President countered by saying more change could be effected from inside the system than outside. He published a plan. Inferred from the moderator’s comment in the second panel/fourth part video, the students had filled the auditorium and she was surprised. This suggests the plan failed, and student activism is possibly coming around again to disrupt the President’s plan. (The first video in that symposium’s series is his speech. I heard nothing in his talk that suggested he had a clue about what is building right under his nose.) Such will be relative chaos. The story that will play out will be out of MIT’s President’s control. But this is simply me speculating a possible story about chaos.

    And in chaos is where we happen to be at this time. Maybe the story aging boomers are living out are the nuclear war drills of our youth. Are we in the metaphorical hall facing the wall with our coats pulled over our heads … waiting for the ‘all clear’ signal!

    Whatever.

    Some of our youngest generation of adolescents do not know this story, or, if they do, will not conflate a former youthful experience with an former adult ones as my father did; as boomers may be doing. They are youth.

    And the youth are on strike for adults to be adults. Academia would be wise to be pulling out all the stops for imagining and [very publicly] implementing a pan-academia business plan that effects zero carbon emissions by 2025 … or reap the whirlwind. There is a story for that … & it is chaos [for the slumbers]; liberation [for the [newly] awoken.

    Vigilant – or not, “We have all been here before!”

    sNAILmALEnotHAIL …but pace’n myself

    https://m.youtube.com/channel/UCeDkezgoyyZAlN7nW1tlfeA

    life is for learning

    so all my failures must mean

    that I’m wicked smart

    >

  62. -1,

    Why does it seem intuitive to you that warming of the planet by a few degrees would only have a relatively minor impact on GDP.

    It doesn’t.

    Does the temperature or rainfall outside affect one’s productivity if they are working on a computer inside an climate controlled office? If I move from Ottawa, Canada to Brisbane, Australia, would my productivity decrease by 90% due to the change in the average local temperature?

    Not everyone works inside on computers.

  63. Ben McMillan says:

    The vision of everyone being rich and working in an air-conditioned building is pretty much baked into models like DICE. I recall that the biggest ‘cost’ of climate change in one of the studies that the DICE damage function is based on is actually the air-conditioning.

    Of course, the natural world is pretty stuffed in a 4C scenario and a lot of people will be living in places that are no longer suitable for agriculture. But that’s OK, cause they’ll be ‘rich’ and they’ll just buy food from Siberia.

  64. verytallguy says:

    Of course, the natural world is pretty stuffed in a 4C scenario and a lot of people will be living in places that are no longer suitable for agriculture. But that’s OK, cause they’ll be ‘rich’ and they’ll just buy food from Siberia

    Precisely.

    The IAMS implicitly assume that such a total overturning of ecology and agriculture can be achieved at zero cost, and that infinite natural resources beyond fossil fuels are available to support it.

    Dr Pangloss would be proud.

  65. izen says:

    @–1=e^iπ
    “If I move from Ottawa, Canada to Brisbane, Australia, would my productivity decrease by 90% due to the change in the average local temperature?”

    If you are working in an office with AC then your productivity may be unaffected.
    However if you live outside the city then bushfires, or heatstroke might compromise your quality of life a little at least.

    https://www.abc.net.au/news/2019-10-06/qld-heatwave-forecast-with-fire-conditions/11575298
    “A wave of dangerous severe fire weather is about to hit southern Queensland with the Bureau of Meteorology (BOM) forecasting heatwave conditions as temperatures soar up to 40 degrees Celsius in parts of the state.”

    There seems to be a common fallacy that a few degrees increase in the climate of a region can be dismissed as insignificant compared to the diurnal and seasonal range of temperature changes.
    Or that it can be equated to moving a few hundred miles closer to the equator and therefore enjoying a slightly warmer climate, the ‘UK becomes like the Med’ error.
    Unfortunately such a shift in climatic region is not accompanied by a similar shift in the physical and biological ecology. The result is at the least disruption of the agricultural environment, and sometimes a rapid and catastrophic transition via wildfires to a new, and much less habitable, barren desert.

  66. The Very Reverend Jebediah Hypotenuse says:

    Number 5 is the weakest link.
    The results won’t make sense even for 2 C warming.

    I belive it was Dawkins who put it this way: There are vastly more ways to be dead than alive.

    The physics of putting things on top of other things works well – until the slope exceeds the angle of repose.
    The human body’s response to a elevated internal temperature is homeostatic – most of the time.
    The economics of fractional reserve banking works well – until it doesn’t.
    Happens all the time.
    You can’t eat stock options or diamonds.

    I haven’t even mentioned the nukes yet.

    Kurt Vonnegut would be proud.

  67. dikranmarsupial says:

    “Not everyone works inside on computers.”

    and not everybody who works inside on a computer does so in a climate controlled office (for instance because they work in a listed building that can’t economically be modernised).

    As usual, if you are wealthy climate change causes little dfficulty, but the options are limited for everybody else. Hard cheese, I’m alright Jack?

  68. Willard says:

    > If anything, two of the main economic IAMs (DICE and PAGE) overestimate the impact of climate change on GDP because they neglect that as countries get richer, their GDP depends less strongly on the local climate.

    So many tricks is so few words. Handwaving to the usual IAMs without showing one’s homework. Alluding to a mere possibility as if it meant anything tangible. Switching from impact on the GDP to impact simpliciter. Formulating an analytical truth as something deeply important – I mean, has anyone ever considered that if GDP depended less on climate it would be less dependent on climate?

    Instead of arguing by stipulation, it might be more profitable to pay due diligence to that dependence relationship. Let’s try something wild – reading a recent paper. Here’s the abstract of one:

    We study the long-term impact of climate change on economic activity across countries, using a stochastic growth model where labor productivity is affected by country-specific climate variables—defined as deviations of temperature and precipitation from their historical norms. Using a panel data set of 174 countries over the years 1960 to 2014, we find that per-capita real output growth is adversely affected by persistent changes in the temperature above or below its historical norm, but we do not obtain any statistically significant effects for changes in precipitation. Our counterfactual analysis suggests that a persistent increase in average global temperature by 0.04°C per year, in the absence of mitigation policies, reduces world real GDP per capita by more than 7 percent by 2100. On the other hand, abiding by the Paris Agreement, thereby limiting the temperature increase to 0.01°C per annum, reduces the loss substantially to about 1 percent. These effects vary significantly across countries depending on the pace of temperature increases and variability of climate conditions. We also provide supplementary evidence using data on a sample of 48 U.S. states between 1963 and 2016, and show that climate change has a long-lasting adverse impact on real output in various states and economic sectors, and on labor productivity and employment.

    https://www.imf.org/en/Publications/WP/Issues/2019/10/11/Long-Term-Macroeconomic-Effects-of-Climate-Change-A-Cross-Country-Analysis-48691

    More on that laters. Life awaits.

  69. Dave_Geologist says:

    Interesting speculation regarding mathematical models. Isn’t it just as probable that the development of computers caused the shift?

    Quite the reverse I’d have said, Greg. Obviously my field is different (geoscience, and, interactively with petroleum and reservoir engineering). So-called “classical reservoir engineering” is equivalent to the analytical rigour and closed-form solutions which are apparently fetishised by economists. It doesn’t need computers. Even Excel is overkill. It uses herds of spatially invariant spherical, elliptical, cuboidal and often two dimensional cattle to estimate the average properties of a complex world that is 3D in both geometry and property distribution. It was developed in the pencil-and-paper days, when a computer was the size of a house and a calculator was a hand-cranked beast half the size of a typewriter. Up until well into my career the SEC still accepted only those methods for Reserves submissions. In part, no doubt, because they could check the working without borrowing a computer from NASA.

    Computers allowed numerical finite-difference models to be built which allowed realistic geometries and property distributions to be represented. You then have to worry about all the numerical things that can go wrong in numerical models, but there are various ways to test for that, including the trusty old analytical solutions which you can benchmark your algorithms against. I always find it amusing that there is an almost exact one-for-one match between GCMs and reservoir simulation models, often run by people who don’t believe climate models, right down to homogenisation-equivalents and parameterisation (called pseudoization). The main difference is that reservoir engineers are much more willing than climate scientists to history match, and to shrug their shoulders when the history-matched parameters start to look non-physical compared to lab results. Go figure.

    In one of my own fields, basin modelling, the canonical paper was McKenzie (1978) SOME REMARKS ON THE DEVELOPMENT OF SEDIMENTARY BASINS. All nice and mathematically tractable, if perhaps somewhat daunting in places, but it only works for a sedimentary basin which is infinitely wide, identical throughout its width, and filled with a material having identical physical properties throughout. Don’t get me wrong, it was a seminal paper and got people looking at extensional basins in a new way. But the sum total of such basins existing in the real world is zero. If you want to do something practical, like find oil and gas, you have to handle real-world basins which don’t have closed-form solutions.

    I published closed-form solutions to slightly more complex geometries in the 80s, comprising repeating identical segments. You can actually answer some useful practical questions using them, but the basin fill still needed to have uniform physical properties so was unrealistic. But I never intended that as an end-point, not even in the initial paper. Just the first half to show that the concept worked where analytical solutions were available, and to introduce the key behaviours of the model (some of which, even with those simplifications, were counter-intuitive). That was handy, because if the counter-intuitive stuff only appeared in F-D models, peoples’ first thoughts would be “artefacts”. But the meat of the paper, and everything I did subsequently along those lines, involved numerical models which required discretisation of the model space and a solver progressing in discrete time-steps. It was the only way to get a useful result, where “useful” ultimately meant something that could contribute to multi-million dollar decisions in the messy real world.

    ISTM that climate economists tend to stop work at the bolded stage, but want us to apply those results to the bold-italics stage.

  70. Joshua says:

    > as countries get richer, their GDP depends less strongly on the local climate.

    […]

    > Handwaving to the usual IAMs without showing one’s homework

    Even at the handwave level, this argument runs into trouble.

    Rather obviously, even without a good faith evaluations of IAMs, this statement *as a universal law* runs into trouble.

    Clearly, countries that depend on the current state of the climate to support their GDP (of which there are many) would ask for more rigorous thinking about handwaving.

    https://mobile.twitter.com/howmuch_net/status/937986464301944832

  71. Dave_Geologist says:

    In a similar vein, doesn’t the idea of perfectly informed agents acting entirely in their own rational self-interest in a perfectly efficient market date all the way back to Adam Smith?

    For fun, another geological example; naturally fractured reservoirs. Classical reservoir engineering can only handle single-porosity, single-permeability and dual-porosity, single-permeability models, not dual-porosity, dual-permeability models. In practice you can use SP-SP if the fracture permeability contributes less than 10% of the total, and DP-SP if the matrix permeability contributes less than 10% of the total. Otherwise you’re stuck, even with numerical simulators, for other than toy models. You don’t even have to go to discretisation: the equations explode and there is no analytical solution. The first practical, real-oilfield DP-DP model I’m familiar with dates from the mid-90s and took a few days to run on a hired cluster of half-a-dozen high-end computers. By the mid-2000s we were running them onsite on a bog-standard Unix server cluster. By the 2010s were were running MonteCarlo simulations on our own supercomputer.

    Again, computers drove us away from closed-form solutions, and let us do things we could never have done with closed-form solutions. There are a lot of reservoirs in that two-orders-of-magnitude gap in the modellable space, and I can think of specific examples which languished for decades because the operator couldn’t model them properly and was scared it would all go horribly wrong in some unpredictable way.

  72. wmconnolley says:

    > doesn’t the idea of … back to Adam Smith?

    No. Try reading WoN.

  73. Willard says:

    > [C]omputers drove us away from closed-form solutions, and let us do things we could never have done with closed-form solutions.

    More than that, computers are shaping the way we think about mathematics:

    “The world of mathematics is becoming very large, the complexity of mathematics is becoming very high, and there is a danger of an accumulation of mistakes,” Voevodsky said. Proofs rely on other proofs; if one contains a flaw, all others that rely on it will share the error.

    This is something Voevodsky has learned through personal experience. In 1999 he discovered an error in a paper he had written seven years earlier. Voevodsky eventually found a way to salvage the result, but in an article last summer in the IAS newsletter, he wrote that the experience scared him. He began to worry that unless he formalized his work on the computer, he wouldn’t have complete confidence that it was correct.

    But taking that step required him to rethink the very basics of mathematics. The accepted foundation of mathematics is set theory. Like any foundational system, set theory provides a collection of basic concepts and rules, which can be used to construct the rest of mathematics. Set theory has sufficed as a foundation for more than a century, but it can’t readily be translated into a form that computers can use to check proofs. So with his decision to start formalizing mathematics on the computer, Voevodsky set in motion a process of discovery that ultimately led to something far more ambitious: a recasting of the underpinnings of mathematics.

    https://www.quantamagazine.org/univalent-foundations-redefines-mathematics-20150519/

  74. Ben McMillan says:

    The key attraction from computer models, from my point of view, is that you get out emergent things that you weren’t expecting, that aren’t just a natural consequence of the assumptions.

    You solve some simple coupled differential equations, and a chaos butterfly appears on your screen.

    You solve fluid dynamics on a sphere and zonal flows and jet streams appear as if by magic.

    IAMs, on the other hand, are linear summative models whose results are not terribly surprising given the assumptions. If you assume that no-one cares about the future (high discount rates) and damage isn’t too bad at 4C, you get out a pathway to a pretty nasty scenario. Then you claim that it is the result of a sophisticated analysis…

  75. izen says:

    @-DaveG
    “doesn’t the idea of … back to Adam Smith?”

    To expand a little on Willards brief reply, the idea of rational individual agents as the basic component of economics is generally ascribed to
    John von Neuman, and Oscar Morgenstern, 1944, Theory of Games and Economic Behaviour.
    Undoubtedly there were precursors, but they get the credit for tying together the philosophical aspect of individual agents and a formal mathematical/logical analysis. Paul Nash expanded the concept of a Game theory equilibrium state in decision making, his paranoia is said to have given him some further insight.

    Adam Smith certainly saw individual tradesmen as important components of an economy, but had a much greater emphasis on the behaviour of the system as a whole – (The invisible hand…) and also considered ‘irrational’ aspects of human behaviour, like sympathy, to be important factors.

  76. Steven Mosher says:

    Still waiting for a better damage function.

  77. Nathan says:

    “Still waiting for a better damage function.”

    How would we recognise it as ‘better’?

  78. izen says:

    @-SM
    “Still waiting for a better damage function.”

    That makes the a prior assumption that the damage, however that is defined, can be calculated via a formalised function that will relate it to other measurable variables.

    It may be that the outcome of various changes in the environment are inherently impossible to project with ANY sort of mathematical, logically consistent function. That while the impacts of one change are contingent, the chain of interactions are iterative and beyond the ability of any formal analysis. That the factors are fundamentally impossible to reduce to metric values.

    The human propensity for constructing a narrative to ‘explain’ how things happen post hoc, is of dubious reliability. The expectation that it can be further reduced to a mathematical formalism seems even less credible. However much we may want to be able to foresee the consequences of current events, they may be intrinsically incalculable.
    Your wait may be for a fictive entity, and potentially futile.

  79. David B. Benson says:

    izen — History shows we can do better than that
    https://en.m.wikipedia.org/wiki/Indeterminism
    although never perfectly. So some sort of “damage function” is moderately predictive.

    For example, southern Vietnam will drown beneath the rising seas over the next 30 years. So now those who care can estimate the number of displacees and speculate as to where they might go. It won’t be to Mars.

  80. Willard says:

    OK, back from the first Chernobyl episode, what better to cheer us up than damage functions. Here’s a recent study shows the importance of selecting the correct optimal path for the whole exercise to make any sense at all:

    We find a big effect of the damage function on the social cost of carbon and a significant impact on the economic growth rate. Using a theoretical framework, this paper shows that the negative impact of climate damages on economic growth grows with the degree of convexity of the assumed damage function. Moreover, contrary to the prevalent opinion in public discourse, optimally designed climate policies do not prevent the world economy from experiencing positive growth rates despite the convexity of damage functions. However, if climate policies aiming to correct the pollution externality are sub-optimally chosen, damages to capital become higher and the economic growth rate may become negative in the long run. Calibrating our model, we confirm that in the social optimum economic growth rates are still positive in the long run even for highly convex damage functions and assuming non logarithmic utility, i.e. for a quartic damage function and a low intertemporal elasticity of consumption substitution. Specifically, a quartic damage function raises the social cost of carbon by more than a factor of ten compared to the linear function, already after 14 years.

    https://link.springer.com/article/10.1007/s10640-018-0219-y

    The short of it is that those who argue that the damage function ought to be linear are better be sure about what they’re saying.

  81. izen says:

    @-SM
    “southern Vietnam will drown beneath the rising seas over the next 30 years. So now those who care can estimate the number of displacees and speculate as to where they might go.”

    I would agree that there are physical changes that can be projected, and the numbers estimated. The social and economic consequences however are not accessible to anything more than speculation. While Mars can be excluded, the cumulative effects of population displacement are not amenable to any quantitative analysis. Or even a qualitative description.
    “’It’s tough to make predictions, especially about the future’” Y.B.

  82. Nathan says:

    If damage functions provide a predictive function, how do they do in predicting the past?
    trying to find anything on this is tricky, Lots of papers on the Little Ice age, but none explicitly detail the GDP ‘cost’.
    Many civilizations have suffered collapse due to climate changes. Those changes in climate do not appear to be of the same magnitude that we’re to experience… So why would the damage be ‘small’?

  83. David B. Benson says:

    izen, I am not SM.

    In the case of Vietnam, the social and economic consequences are easy enough to predict. Almost all of the people will leave the south and whatever livelihoods they have there.

    What only experts on Southeast Asian affairs might speculate about with some accuracy is where everyone will go.

  84. David B. Benson says:

    Nathan, climate change has also given rise to civilizations. The most remarkable is Pharaohic Egypt, due to the sudden drying of the savanna to the west of the Nile River, giving rise to the Sahara desert and forcing all the inhabitants to become farmers along the river.

  85. Nathan says:

    David
    Sure, but during the transition, from hunter gatherer to farmer (I assume that’s what happened) – how did people fare? Was this a good experience? And what was change in temp and the change in GDP? I fond it hard to reconcile how a change in temp leads to a change in GDP in any linear sense.

    And that also was a case where there was somewhere to go. Most land is now used for Ag or industry, there’s not many places to move to that aren’t already inhabited.

  86. izen says:

    @-David B. Benson
    “izen, I am not SM.”

    My apologies. I type my replies in notepad and failed to change the header.

    @-climate change has also given rise to civilizations. The most remarkable is Pharaohic Egypt, due to the sudden drying of the savanna to the west of the Nile River.

    The Sahara got monsoon rains from ~8500BC till ~5300BC
    There was an agrarian/pastoral civilisation that extended into this area from the Nile valley and then migrated back.
    It is open to debate whether that 3000 year transition triggered the Egyptian Pharohic era, or if the rise of other civilisations that initiated trade, conflict, and cultural exchange were more important factors. The pre-pharohic societies may have been more diverse than the constrained locality seems to have engendered in the Nile valley.

  87. David B. Benson says:

    Nathan and also izen — I am no expert. From what little I know farming by “the sweat of thy face” was a cursed, forced response to climate change. The anthropologists keep telling me that hunter-gatherer societies have happier people who have plenty of time for activities not directly related to obtaining food.

    As for the Pharoahs, as other agricultural civilizations, the upper crust can lord it over the peasants because they can’t simply leave and strike out on their own. That’s how the pyramids were able to be constructed in Egypt and Mesoamerica; so I opine.

  88. Nathan says:

    Is this a better estimate?
    ftp://ftp.repec.org/opt/ReDIF/RePEc/bcu/papers/iefewp86.pdf

    How would this rate?

  89. Nathan says:

    David

    Yes, and slavery is no fun thing!
    One of the problematic things of using GDP is that it’s not directly related to ‘good times’ – pretty sure GDP went up during WWII…
    generally Humans will seek ‘good times’ not ‘Higher GDP’

  90. David B. Benson says:

    Pharaoh, tch, tch.

    Yes, GDP is a wrong measure.

  91. izen says:

    @-Nathan
    “Most land is now used for Ag or industry, there’s not many places to move to that aren’t already inhabited.”

    That is not quite correct. The UK is one of the more heavily populated Nations, it has ~70% of the land under agriculture and ~6% urban/industrial.
    So nearly a quarter is open for further use without disruption of current occupation or production.
    England with a population density several times greater than the other regions still has just under 3000 sqr yards per person.

    Hunter-gatherer societies may be better fed with less effort. But there is no surplus that precludes any social development or diversity. Flint tools and the bow and arrow is about as far as we got for ~180,000 years.
    Agriculture does tend to engender slavery -> God/king hierarchies, but does enable bronze/writing/iron/astronomy/weaving/ceramics…

    There are at least ten civilisations that collapsed from drought.
    https://www.wunderground.com/blog/JeffMasters/ten-civilizations-or-nations-that-collapsed-from-drought.html
    To this list the West Saharan, Harrapen, and Olmec can probably be added. The role of increasing population exceeding its water supply probably exacerbated the effects of drought in most cases.

  92. Ben McMillan says:

    The idea is that ‘waiting for a better damage function’ or ‘some bright spark making a better IAM’ is misguided. The whole cost-benefit approach is inappropriate, and ill-posed. Too many questions that can’t be answered, about what the costs are, and how much we should care about other people, so it ends up giving any answer you want.

    Wiping out the majority of other species on the planet will have an effect that echoes down through the ages, but is less important in an IAM than the cost of some more air conditioning for a couple of decades. Is economics really able to definitively make this judgement call?

    Instead, why not adopt an ‘engineering’ type approach: life-critical systems must be designed with an appropriate safety margin and with sensible fall-backs.

    Basically, it looks like we have a serious risk of widespread impact on systems crucial to human survival at 3C. So let’s make sure we don’t end up there, and we had better leave a safety margin. If you like, take the idea of ‘planetary boundaries’ seriously.

    That’s basically equivalent to a brick-wall damage function at 2C; probably practically equivalent to just using some high order polynomial like a quartic.

  93. Nathan says:

    Izen

    I’m not really disputing your numbers, but this:
    “That is not quite correct. The UK is one of the more heavily populated Nations, it has ~70% of the land under agriculture and ~6% urban/industrial.”

    is ‘most’ – and recall the transition from hunter gatherer to agricultural was one with numbers FAR lower for ag.

    It also raises the question of how much land should we reserve for ‘nature’ or wild country. Very little of that left – mostly because we have no way of assigning it a dollar value as ‘wild’

  94. Dave_Geologist says:

    Thanks to those who enlightened me on The Enlightenment. The Invisible Hand seemed to me hand-made (if you’ll pardon the pun) for cold, hard maths. Oddly enough, I do have the Wealth of Nations in my e-library. Perhaps I should read it 😦 .

  95. Dave_Geologist says:

    Nathan, a lot of the 24% is in Scotland, and looks like this

  96. -1=e^iπ says:

    ATTP –
    “Not everyone works inside on computers.”

    No, but many people in a modern society do. Or they work inside factories, inside hospitals, inside universities, or inside restaurants. The part of the economy that is more strongly dependent on the climate (such as agriculture) becomes a smaller and smaller part of overall GDP as countries get richer.

    Willard –
    Thanks for the study link. I’ll have to look over this recent want. In the past, I have had some concerns with temperature-growth studies, but hopefully this new study will alleviate my concerns.

    – Such growth studies often include a model where economic growth rates of countries diverge as time approaches infinity. This is not consistent with the observation of technological spill-over between countries and less advanced countries learning from more advanced countries.
    – Sometimes these studies have statistical issues. Burke et al. 2015 neglecting stationarity is an example.
    – It seems bizarre to me to talk about the impact of temperature or climate on economic growth rates, but not talk about the impact of energy prices on economic growth rates. Energy is a direct input into the production of new technologies, and increasing the price of energy (through climate mitigation policy) might have adverse effects on technological progress. Example: if it is more expensive to simulate protein interactions on a super computer, then this would lead to less computer protein simulations, resulting in a slower development of new drugs (and then perhaps some people dying due to lack of access to these drugs).

  97. -1,
    You still seem to be ignoring that in many parts of the developing world people don’t work inside and it seems unlikely that within a generation that this will change sufficiently so that they are not impacted by the effects of climate change.

  98. izen says:

    @–1=e^iπ
    “It seems bizarre to me to talk about the impact of temperature or climate on economic growth rates, but not talk about the impact of energy prices on economic growth rates. …Example: if it is more expensive to simulate protein interactions on a super computer, then this would lead to less computer protein simulations, resulting in a slower development of new drugs”

    What is truly bizarre is an argument that the energy required to simulate protein or any biochemical structure and effects by computer modelling is in any way a significant percentage of the cost of the drug.
    FAR more energy is going to expended on the advertising and marketing of a drug. The 3D rendering of CGI in adverts is going to consume an order of magnitude more energy than drug design. And then there is the AC at work, travel and home that everyone is going to need in increasing amounts… whether they are drug designers or marketing executives. Perhaps the development of drugs could be given a slightly higher priority/energy quota to compensate? The ratio of distribution is not fixed.

    The argument also fails on the fact that renewable energy sources are now undercutting fossil fuel sources, which is why coal is disappearing as a energy source. The claim that ‘Green’ energy is more expensive is getting less credible by the day. Not surprising when you have an energy source that does not have any fuel costs.

  99. -1=e^iπ says:

    “You still seem to be ignoring that in many parts of the developing world…”

    I don’t understand why you would think I am ignoring the developing world. I mentioned the empirical finding that GDP becomes proportionately less dependent on climate as GDP increases. For an agrarian society, climate would be very relevant.

    On the topic of the developing world, a lot of people in China, for example, work indoors in factories. That’s why so much stuff is made in China.

  100. -1,
    Okay, let me rephrase. You seem overly confident that we will modify our working practices suitably in all parts of the world that are likely to be affected. I’m not nearly as confident.

  101. Willard says:

    > I mentioned the empirical finding that GDP becomes proportionately less dependent on climate as GDP increases.

    That’s certainly one way to spin it. Here’s a para from K19, op. cit., taken from section 3.2, Long-Term Impact of Climate Change on Economic Growth:

    Most studies in the literature provide evidence for the uneven macroeconomic effects of climate change, with adverse short-term consequences in countries with hot climates, such as low-income countries; see, for instance, Sachs and Warner (1997), Jones and Olken (2010), and Dell et al. (2012). In other words, when a rich (temperate) country is warmer, there is little impact on its economic activity. There are intuitive reasons and anecdotal evidence for this, including adaptation that has taken place particularly in advanced economies; they are more urbanised and much of the economic activity takes place indoors. For instance, Singapore has attempted to insulate its economy from the heat by extensively engaging in economic activity in places with air conditioning. Therefore, if individuals are aware of how extreme heat affects their economic performance, they can invest in self protection to reduce their exposure to such risks. More recently Burke et al. (2015) and Mejia et al. (2018) also show that the negative short- and medium-term macroeconomic effects of climate change are more concentrated in hot countries (i.e. mostly low-income countries).

    That GRRRRROWTH could be decoupled from CO2 trivially follows from the fact that we now take spiritual products into account. The possibility of a purely spiritualist society, however, remains to be seen. If absolute decoupling from CO2 is actually in the cards, then it is to wonder why we’re having this discussion – let’s just stop dumping the stuff into the atmosphere like there’s no tomorrow right here right now. If Minus simply suggests that we’ll adapt, then it may be more expedient to simply say “but we’ll adapt” and stop wrapping the slogan into abstruse armchair economizing.

  102. David B. Benson says:

    What is the damage function of very bad air?
    https://www.bbc.com/news/world-asia-india-50280390

    I was in northern India in July of 1979. Then the air was fine.

  103. Joshua says:

    -1 –

    > I mentioned the empirical finding that GDP becomes proportionately less dependent on climate as GDP increases

    Except that isn’t true. It depends on the source of the increase in GDP growth. If the increased GDP stems from something like tourism the it would reflect greater dependence in climate.

    Your statement is meaningless without the appropriate caveats.

  104. Joshua says:

    – 1 –

    > “It seems bizarre to me to talk about the impact of temperature or climate on economic growth rates, but not talk about the impact of energy prices on economic growth rates….

    It seems bizarre to me to talk about the impact of energy prices on economic growth rates without talking about the relationship between price and cost – in other words without talking about whether price reflects the net balance of positive vs. negative externalities.

    Well, actually not bizarre. It is such a frequent phenomenon that bizarre won’t work. Banal is perhaps a better adjective.

  105. -1=e^iπ says:

    ATTP –

    “You seem overly confident that we will modify our working practices suitably in all parts of the world that are likely to be affected.”

    Where did I say or imply this? The question of this thread are if economic impacts of a few degrees of warming could be a few percentage of GDP, as suggested by some IAMs. I’ve said that this is consistent with observations about society, but I don’t know how this would make you make the above statement.

    Willard and Joshua –

    I don’t know where you are coming from in your claims. I support a pigouvian tax to internalize externalities.

  106. ecoquant says:

    A lot of writing and thought in the above! I saw the post, but did not get a chance to ready any of the details until late this evening.

    I have two thoughts, one a general one, and the other regarding the “fat tail” of climate sensitivity.

    General: Perhaps it might be useful to posit an alternative risk evaluation. Consider what might be the economic risk of an initially limited nuclear exchange between moderate nation states. I like that consideration because the “runaway effect” is clearly a possibility, is non-linear, and it has long term consequences for future economic activity, namely, that some portions of the globe thereafter will be places where no one will wish to pursue economic activity. Can IAM-like approaches assess such a thing? If not, why not? And if they can’t, what makes climate disruption different so they have confidence they can. And if they can assess the impacts of a limited nuclear exchange, what might they be? And how do they compare with projected IAM impacts of climate disruption?

    Fat tails: There is an operational and experimental definition of what constitutes climate sensitivity, in terms of units of doubling atmospheric CO2 concentrations. Setting aside for the moment that this, in itself, makes a bunch of assumptions regarding the climate system, the theoretical expression for climate sensitivity which the operational one tries to measure is pretty complicated.

    R. Knutti, M. A. A. Rugenstein, “Feedbacks, climate sensitivity and the limits of linear models”,

    Essentially, the climate sensitivity function has, in its current examination, been linearized about the present point, and an estimate taken, using various clever experimental means, of what its value is. The point is these cannot tell us anything about departures from linearity. We barely have a sense of what transient or equilibrium sensitivity is, let alone how it might have varied through paleohistory. Current estimates, as clever as they may be, also involving making certain important assumptions.

    Professor Pierrehumbert has, with co-authors, repeatedly emphasized these complexities, ones which students of the problem should consider.

    R. T. Pierrehumbert, “Hot climates, high sensitivity”, PNAS, 27 Aug 2013, 110(35).

    J. Bloch-Johnson, R. T. Pierrehumbert, D. S. Abbort, “Feedback temperature dependence determines the risk of high warming”, GRL, 42, 4973-4980, 10.1002/2015GL064240.

    Indeed, Bloch-Johnson, et al state the pertinent:

    In particular, the linear approximation breaks down for the long tail of high-sensitivity cases, which play a large role in assessments of economic risk [Weitzman, 2011]. We show that observational estimates of climate sensitivity likely underestimate the risk of high warming by neglecting equilibrium nonlinearity,and that understanding this nonlinearity is essential for reducing our uncertainty of these high-risk scenarios.

    One companion question is how quickly can such departures from linearity be realized?

  107. -1,
    Okay, maybe you need to be a bit clearer. You seem to be saying that this is what will happen. If all you mean that this is what *could* happen, then sure, but that not only implies that we can indeed adapt on a suitable timescale, but also that we’ll actually do so. Neither of these are obviously true.

  108. Chubbs says:

    Might be useful to review the past 10-15 years or so to judge how useful IAMs are likely to be in forecasting 2100 conditions:
    1) Temperatures spiked post-hiatus and many climate impacts are appearing faster than anticipated
    2) Fossil fuels are losing competitive advantage, particularly coal, while nuclear/CCS have languished.
    3) Globalism is being questioned. Nationalism is on the rise.

    My crystal ball is pretty hazy for 2030, much less 2100.

  109. ecoquant says:

    I didn’t completely finish the post last night, before needing to sleep.

    The Knutti-Rugenstein paper is here. The page from Professor Pierrehumbert’s text having the expression for ECS is:

    testifying to the complexity of that expression and its non-linearity.

  110. Joshua says:

    -1 –

    > I don’t know where you are coming from in your claims.

    It’s kind of a pet peeve of mine when people talk about the impact of energy “price” without talking about the relationship between price and cost. (Even though no one else seems to care about it very much).

    Consider a hypothetical country which produces fossil fuels, and where the price is extremely low because providing access is easy; however, extracting those fossil fuels is done by slave labor, results in a high level of environmental damage, and results in high levels of particulate matter in the air.

    In such a case, it seems to me to be rather shallow to theorize about the impact of the low energy price without considering the associated costs. I think that conflating price with cost is problematic.

    Seems to me that I often read “skeptics” talking about the (negative) economic impact of higher energy prices without incorporating some consideration of the costs associated with lower prices. Sometimes, they even talk about the benefits of lower prices, without considering costs, even as they (1) talk about the positive externalities associated with access to energy and, (2) when asked about negative externalities, explain that assessing externalities is just too hard to do. Seems to me that you can’t really assess the impact of energy prices robustly if you haven’t at least attempted to address the impact of both positive and negative externalities.

  111. -1=e^iπ says:

    ATTP – “You seem to be saying that this is what will happen. If all you mean that this is what *could* happen…”

    I was more referring to what the best available evidence suggests. Obviously you should take into account uncertainty and have some risk aversion. William Nordhaus, winner of the Nobel Memorial Prize in Economics for his work in climate economics, does this very well in his IAM. For example, he recently accounted for the probability distribution of possible “tipping points” or more accurately instabilities in the Greenland Ice Sheet. He found that it only had a minor impact on the social cost of carbon and the pigouvian level of taxation.

    https://www.pnas.org/content/116/25/12261

    If you think there are missing “tipping points” or instabilities that IAMs are not accounting for, then it would be more productive to quantify the probability distribution of these instabilities and then get them in IAMs. This would be more productive than what I see some people do, which is use a gap-of-IAMs type argument (similar to god-of-the-gaps type argument creationists use) to try to ignore IAMs and justify extreme levels of mitigation.

  112. -1,
    Except we’re moving into an environment we’ve never experienced before. So, how do you develop an IAM or, more correctly, a damage function that you can test? Weitzman, for example, argues that one could use an exponential damage function that would, for small changes in temperature, be consistent with the quadratic one that is commonly used. How do you determine which one of these damages functions is appropriate for the relatively large temperature changes that we are likely to experience and have never experience before?

  113. David B. Benson says:

    In general, I opine, the logistic function, of just two parameters, offers a reasonable approximation to reality.

  114. Greg Robie says:

    @-Dave_G – & an aside – I am very appreciative of your decision to not remain a lurker. The insights you contribute, and their professional context, often informs in ways my blue collar biases appreciate. In this case, that critique of economists’ fetishes with closed-form solutions.

    FWIW, I heard @- izen associating the fears engendered from experiencing the economic crash of ’29 with the attractiveness of [ostensibly non-emotional] mathematically derived structures for describing economics. My comment about computers was predicated on McLuhrin’s insight that “the medium is the message”, and not intended to discount your point. And, BTW, my great grandfather, a banker (& dead set against the 1913 Federal Reserve Act) was out of the stock market when it collapsed in 1929, in part because he saw banking differently; as a social trust. So perhaps all three points of view have relevant in terms of our being creatures of stories.

    Anyway, I’m pretty sure closed-form solutions are similar to what is called single-loop learning in this graphic:

    https://www.researchgate.net/figure/Single-double-and-triple-loop-learning-as-part-of-climate-change-adaptation-Source-IPCC_fig2_290445325

    If so, such informs for me a “why” concerning the critique Nordhaus’ DICE work has been subjected to in this comment thread: it is limited to single-loop thinking, or learningNOT!. Such thinking is all well in good if the past is the future, but everything I am aware of concerning the change unfolding in the climate is that it is transformative.

    Doesn’t this mean that the context of everything is changing?

    @-ATTP, I dismissed the 5 listed options as delusional without showing my work which [I feel] supports the assertion. So let me start with what I would call a know risk factor of a 100% possibility: the collapse of the current iteration of human civilization. Collapse is the one constant of anthropogenic history. To dismiss this is to both not learn from history, AND repeat it (or, so the story goes!). 😉

    As @-izen documented, at least 10, likely 13 civilizations exited over drought. An argument that Rome imploded due to a change in the weather shifting populations south causing Rome’s means of maintaining an empire to fold is one I experience as well made. Hasn’t a failure to see and adequately respond to threats done in every human effort at a civilization 100% of the time? The likelihood of this being a function of motivated reasoning is, as far as I can see, pretty high. So isn’t economic modeling that is based on predicted responses systemically flawed … when higher orders of learning are not factored in (assuming the species has evolved to socially do this kind of learning!).

    I observe that trusted feelings about predictive economic models that ignore history are the problem, and thereby, irrational/delusional. I also argue that this irrationality reveals a mostly hidden motivated reasoning and means that what is functionally ‘religious’-as-delusional is the engine for the problem. To the degree our global civilization of GREED-as-go[]d’s CapitalismFail is locked in with single loop learning the current [consumable] economic analyses: #1 – do not properly reflect overall impact; #2 – are not reasonable; #3 – is laughable given the global dependency of the high consumers’ trusted homeostases (my biggest brook trout were the first – and only – ones to expire this summer as water wained on this hillside); #4 – it is transformative systemic impacts, as substantive, that assign inadequacy to the trusted analyses; #5 – & to the degree the Inuit observations and the omitted diminishing heat storage capacity of the latent heat of Arctic sea ice are committed warming factors, isn’t it looking like 1°C was right all along (distracting Nobel Nordhaus (& a social need for hope) withstanding)? Don’t “proper” economic analyses have to include assumptions concerning the inevitability of collapse as an outcome to be worth the time spent coding them?

    sNAILmALEnotHAIL …but pace’n myself

    https://m.youtube.com/channel/UCeDkezgoyyZAlN7nW1tlfeA

    life is for learning so all my failures must mean that I’m wicked smart

    >

  115. Ben McMillan says:

    Nordhaus’ ice sheet dynamics paper is worth a look: I musn’t be alone in thinking that is is a reductio-ad-absurdum of applying an IAM to long-timescale damages.

    Does anyone else want to guess what happens when you assume a 4% discount rate and look at a process that takes thousands of years?

  116. Steven Mosher says:

    “I don’t understand why you would think I am ignoring the developing world. I mentioned the empirical finding that GDP becomes proportionately less dependent on climate as GDP increases. For an agrarian society, climate would be very relevant.”

    agriculture labor ( I assume thats largey outside) is down to 28% of the total labor force.
    canada? 1%, denmark 2%, China 27% Hong kong 0, UK, USA 1%
    falling everywhere

  117. “agriculture labor ( I assume thats largey outside) is down to 28% of the total labor force.”

    I did a short stint in marketing for an ag co-op right after college. I remember profiling a farmer in his 60s who raised soybeans on several hundred acres without a single employee. He told me farming is all about being a good money manager, diesel mechanic, and knowing how to weld metal is a big plus. The guys running around outside raise cattle or sheep (pigs and chickens are largely indoors in genuinely awful smelling facilities. Dairy farms smell great and are largely indoors, but the hours suck)
    The profile wasn’t about his lack of employees- that was common – it was about how he was using GPS in the cabs of his tractors and combines to increase yields. Those cabs are fully enclosed, air conditioned, heated and every bit as comfortable as a nice car.

  118. ecoquant says:

    @0=1+e^{i\pi},

    If you think there are missing “tipping points” or instabilities that IAMs are not accounting for, then it would be more productive to quantify the probability distribution of these instabilities and then get them in IAMs.

    Ha! Facts are that when speaking of these instabilities, particularly bifurcations, our science is too weak to be able to predict them. (“Local analysis provides no information about what is on the other side of a bifurcation.”) In other words, while a bifurcation may be possible, we don’t really know where they are, and if we were approaching such a place, we wouldn’t see it coming. Accordingly, creating a probability mass distribution of these is simply not feasible.

    Lecture at:

    https://www.pathlms.com/siam/courses/3040/sections/4272/video_presentations/31393

    Slides at:

    Click to access Pierrehumbert_IP4_5-23-11_2pm.pdf

    A survey of the subject by Dijkstra is available here. And there are some recent papers:

    Dekker, von der Heydt, Dijkstra, “Cascading transitions in the climate system“, 2018
    van der Bolt, van Nes, Bathiany, Vollebregt, Scheffer, “Climate reddening increases the chance of critical transitions“, 2018

    [F]rom physics … the actual coefficients that enter into the climate system … are very smooth functions of amount of Carbon Dioxide. Any sort of switches or discontinuous behavior, bifurcations, complexity does not from the dependence of the coefficients in the equation upon Carbon Dioxide, it comes from the collective response of the system to how Carbon Dioxide changes the energy budget.

    (Professor Pierrehumbert, at about 11 minutes into the talk)

  119. Willard says:

    > Does anyone else want to guess what happens when you assume a 4% discount rate and look at a process that takes thousands of years?

    Near zero:

    [T]he most important single indicator of the strength of current climate policy is the SCC. Therefore, a useful way of understanding the impact of GIS disintegration on climate policy is to estimate how much adding GIS damages or GIS volumetric constraints changes the SCC. This study demonstrates that, under a very wide range of assumptions, the risk of GIS disintegration—although a major change in the earth system—would make a small further contribution to the overall SCC or to the overall cost of climate change. The increment to the SCC is near zero at moderate discount rates and as high as 5% of the total SCC at very low discount rates and high melt rates. At the discount rate used by the US government, the addition of GIS damages to the SCC is essentially zero. The intuition behind this result is that the timescale of damages associated with GIS melting is much longer than those associated with non-GIS damages. For example, impacts on agriculture are determined largely by contemporaneous climate changes, while GIS melting is determined by climate only with a long lag.

    https://www.pnas.org/content/116/25/12261#sec-10

    Near the end of the SI there’s an interesting table, showing that discount rates have the same effect on the Social Cost of Carbon (SCC) as changing melt rates and volume constraints:

    https://twitter.com/nevaudit/status/1191751769065426945

    With his first choice, Bill lets Greenland melt by 90%. Then he finds it has a low SCC. Well, duh.

    Not sure I’d bet the farm over Bill’s intuitions.

  120. dikranmarsupial says:

    Steven Mosher wrote “agriculture labor ( I assume thats largey outside) is down to 28% of the total labor force. canada? 1%, denmark 2%, China 27% Hong kong 0, UK, USA 1% falling everywhere”

    [Joyce Grenfell]
    … all very well, my dear, but who will clean the drains?
    [/Joyce Grenfell]

    Not all jobs can be performed in an air conditioned office. I would have thought market forces would mean that those who end up doing the unpleasant outdoor jobs will end up being very highly paid as nobody else will be willing to do that (but history suggests that the labour market isn’t like that in practice).

  121. The Very Reverend Jebediah Hypotenuse says:

    Adam Smith, Adam Smith, riding through the land…

    I live near several modern croping farms of thousands of acres each.
    Their profitability is dependent on many factors: scale i.e. monocropping huge swaths of land, relatively cheap fossil fuels, fertilizers manufactured through the energy-intensive Haber-Bosch process, glyphosate-resistant crops and their attendant applications of glyphosate, low land tax rates, agricultural subsidies, and the importing of cheap foreign labour during harvest.
    And a stable regional climate.
    Farm backruptcies are increasing, not decreasing.
    This is the human food chain.

    Anyway, by 2050, which, according to my careful calculations, is just over 30 years from now, sea levels will rise to the point where quite a few major cities will be underwater during high tides.

    We’re talking Miami. We’re also talking Bangkok, Shanghai, Mumbai, Alexandria, and most of Southern Vietnam.

    Aside from any ecological and economic damages that may or may nor be modeled by our models, that’s at least 150 million people that will be looking for a new place to live.
    Within 30 years.
    Now that’s a migration crisis.

    Meanwhile, we can witness the extirpation of birds and the plunge in pollinating insect populations. Off the coast, coral reefs are decimated and kelp forests are dead or dying because of high marine temperatures, which affects the plankton and molluscs and the sea urchins and the sea-stars, which then affects the fish, which then affects the fisher-people, and, gosh, this is getting mighty close to that human food chain that supplies you and me.

    Meanwhile, the fire upon fire washes across California wine country.

    Meanwhile, the USA is preparing to pull out of the Paris Accord.

    Meanwhile, the Keeling curve continues to climb to heights not seen since the mid-Pliocene, 2 to 4 million years ago. Which is, coincidentally, about when humans began to learn how to burn stuff.

    Smith himself regarded ‘The Theory of Moral Sentiments’ as a more important work than ‘Wealth of Nations’. Capital economies are easy to understand relative to the “mutual sympathy of sentiments” that Smith argues is the source of human conscience.

    I think maybe young Greta and her friends get all this in ways that older folks don’t or won’t.
    Small wonder they’re angry, and not that much interested in better damage functions, reasonable discount rates, or gee-whiz, GPS-driven, climate-controlled combines.

    Now – Hand over all your lupins.

  122. ecoquant says:

    @dikranmarsupial,

    If we continue our collective inaction long enough, it’s entirely reasonable to expect certain critical outdoor jobs for some parts of the year will need to be done in suits having portable A/C in them. That’s because there will be places and times when it will be too hot for people to survive outdoors. Exposure to 35C for extended periods is not survivable. This will occur if we see a +7C increase of temperatures on land, which corresponds to a +4C to +5C increase in global mean temperatures.

    I don’t believe IAMs have considered that kind of structural change.

  123. dikranmarsupial says:

    ecoquant, hopefully not, but I suspect they will still be performed by people on minimum wage or there about.

  124. Willard says:

    > I don’t believe IAMs have considered that kind of structural change.

    The DICE-GIS-v32-standard-1111-base.gms file contains 435 lines:

    https://dataverse.harvard.edu/dataset.xhtml?persistentId=doi:10.7910/DVN/UYCPFL

    About half of them are parameters, and a third are variable declarations. There is one loop, and it is 54 lines long.

    I had to search to get that what Bill advertizes as the DICE discount is more than 4%.

    By way of contrast, Olga Tokarczuk, the current Nobel Prize in Literature, wrote more than 30 books. Her most recent one, **Drive Your Plow Over the Bones of the Dead**, contains more than 300 pages.

    And then people say that arts and humanities are not competitive.

  125. ecoquant says:

    @Willard,

    I don’t see anything in that base code which is a step change or anything. Assuming Social Cost of Carbon is the quantity that assesses impact, it’s a straight ratio of two others.

    Of course, I don’t understand any of this deeply, but this does appear to be like just a big linearization about the present, and does not in that form reflect what might happen if regions of the United States begin to be starved of important staples.

  126. Willard says:

    > this does appear to be like just a big linearization about the present

    There are dynamic components: ice melt is simulated using hysterisis functions. But you’re right – in the end, it looks more or less like a compound interest calculator.

    Part E of the SI describes the modeling simplifications and adjustments.

    Suffice to say for now that producing the Most Important Model in the World in proprietary languages looks underwhelming to me.

  127. Sinocentric says:

    The arguments I find most persuasive to square the circle of “existential threat” and “10% GDP loss” are:

    1. Growth may not continue at 2% throughout the 21st century. This seems quite plausible. Nothing can grow forever. Consider this: most industrialized economies are on a slowing growth trajectory. They are also aging. The countries which will be most damaged from climate change are also those with the most obvious room for development and those with younger populations. This could really undermine future growth estimates. Perhaps we will see a 10% GDP loss from 4 celcius, but if our growth is lower than projected, this is more damaging.

    2. Society will react with feedbacks that can damage the economy further. Conflict over low wages, missed expectations, inequality, immigration, or resources seems the most obvious one. Considering how negative our politics are over these issues already makes that seem likely.

    3. Some people overplay Climate Change to an extent. The world probably isn’t going to end in 10 years in an acute climate event. This sort of muddies the waters in terms of expectations. Very bad things will happen, but perhaps not on the scale or timeline people are imaging.

    On the other hand,

    4. We are missing something. Maybe there is a huge “tail risk” we are unaware of that could bring it all down. All things considered it would be better to not chance this, but, here were are. lol.

    Just my opinion,
    Kev

  128. -1=e^iπ says:

    @ ATTP –
    “Except we’re moving into an environment we’ve never experienced before.”

    We’ve already warmed 1 degree. That’s quite a significant change and gives us information on what would happen if we warmed another degree or two.

    More importantly, we can use variation in local climate over time, which is larger, and the large variation in local climate between different regions of the planet (say between Canada and Australia) to estimate the impact of climate change on the economy. This is what the empirical literature does.

    “How do you determine which one of these damages functions is appropriate for the relatively large temperature changes that we are likely to experience and have never experience before?”

    Obviously there is a difference in the third derivative of impacts with respect to temperature, thus the different models have different empirical predictions, which can be tested against the data.

    And with respect to disagreements between people who support the Paris Agreement, and those who support a pigouvian tax justified by IAMs, the difference in long term climate change between the two policies is only about 1 degree C, so with respect to arguing against a pigouvian tax, higher derivative beyond the quadratic are moot.

    Also, I find it odd that those arguing that we can’t extrapolate results of IAMs to the future will often point to these growth & climate studies and then make much larger extrapolations. Seems like a double standard to me.

  129. kev opines: “3. Some people overplay Climate Change to an extent. The world probably isn’t going to end in 10 years in an acute climate event. This sort of muddies the waters in terms of expectations. Very bad things will happen, but perhaps not on the scale or timeline people are imagining.”

    Mike says: it’s good to be careful and precise with our language regarding climate change. The world is not going to end due to a climate event, acute or otherwise. The sixth extinction, however, is going to increase in breadth, depth and severity and the potential for extinction of many, many species in the next 50 to 100 years is very high. It is foolish for any species to think that it will probably do ok in a great extinction event. The world will not end, but many species will and there is likely to be a great deal of suffering that could be reduced if we took this problem as seriously as we should and actually embarked on a global, all-out attempt to cut emissions very, very quickly. To do this, we would need to see nationstates and communities decide to not engage in the prisoners dilemma game, but to instead commit immediately to reduce emission starting right now.

    It seems unlikely that our species will do what it needs to do to address the problem of global warming and that’s a real shame. As time passes and more ghg accumulates in atmosphere and ocean, our efforts become less and less about a real effort to slow the great extinction event and more about living in a graceful manner. Living gracefully as individuals, living as if our lives and actions matter, is the next best thing to changing the way we live and actually addressing the problem on the level of our species.

    Just my opinion on that as well,

    Warm regards

    Mike

  130. Steven Mosher says:

    “Part E of the SI describes the modeling simplifications and adjustments.

    Suffice to say for now that producing the Most Important Model in the World in proprietary languages looks underwhelming to me.”

    haha look at you skeptics rummaging through code. Imagine a world where Nord
    refused to release his source because it would be better science if you replicated it

  131. Willard says:

    > look at you skeptics rummaging through code.

    All eight of us:

    https://twitter.com/nevaudit/status/1191789136811835394

  132. Ben McMillan says:

    For me, the amusing thing in Nordhaus’ paper is thought of the humans and their cities slowly migrating up hills for a couple of thousand years as they allow the sea to rise (because they are capable of planning only on a 20 year timespan).

    From a sci-fi point of view, does that mean an array of deserted skyscapers sitting out in the ocean next to New-New-New-New York, or island-cities, far from the coast, built on the ruins/garbage of previous civilisations?

  133. David B. Benson says:

    Ben McMillan, the waves will quickly demolish all the structures.

  134. Steven Mosher says:

    “From a sci-fi point of view, does that mean an array of deserted skyscapers sitting out in the ocean next to New-New-New-New York, or island-cities, far from the coast, built on the ruins/garbage of previous civilisations?’

    No, the sea level rise is not predicted to happen overnight. If folks are smart they will employ a combination of measures.

    1. Sea walls where it makes sense
    2. Managed retreat in other places.

    SFO plans to surround airport with 10-mile wall to protect against rising bay waters

    Meh, we knew this shit long ago

    https://www.scientificamerican.com/article/tidal-gate-across-san-francisco-bay-proposed-to-manage-sea-level-rise/

  135. -1,

    We’ve already warmed 1 degree. That’s quite a significant change and gives us information on what would happen if we warmed another degree or two.

    In terms of testing our understanding of climate change, yes. In terms of testing how society responds, I’m not sure this is true. Most people expect the impacts to grow non-linearly, so it’s not obvious that the impacts for small changes are necessarily a good guide of what they will be if we warm much more.

    More importantly, we can use variation in local climate over time, which is larger, and the large variation in local climate between different regions of the planet (say between Canada and Australia) to estimate the impact of climate change on the economy. This is what the empirical literature does.

    Yes, but some people argue that this isn’t a good empirical way of estimating how we will respond to similar levels of *global* warming. I must admit that I find this surprising, because it’s not obvious to me that using variations across the planet today are a good guide of how society will respond to variations due to future climate change. These do seem like somewhat different things. Australia and Canada, for example are different. There are many reasons why they have different economies. It’s not clear to me at all that looking at these variations today is a good guide of how economies will change as regions warm under global warming.

    Also, I find it odd that those arguing that we can’t extrapolate results of IAMs to the future will often point to these growth & climate studies and then make much larger extrapolations. Seems like a double standard to me.

    I’m not sure what you mean here, but I’m not saying you can’t extrapolate them. I’m pointing out that we’re extrapolating into a region of parameter space we’ve never experienced before. Hence, we have to trust that our empirical estimates, either based on the smaller changes we’ve already experienced or the empirical estimates based on local variations today, are suitable for this type of extrapolation.

    As far as I’m aware, there is more than one non-linear function that would fit the data we have today, but that would produce wildly different extrapolations. How do we have confidence that the one we’re using (mostly quadratic) is actually the most reasonable.

  136. Steven Mosher says:

    “All eight of us:”

    That’s on par with skeptics who looked at hansens code ( 3 of us)

  137. Ben McMillan says:

    Well, sure, it probably just all gets pounded to rubble (the plinth below the statue of liberty should keep her feet dry for quite some time).

    But the structures are going to have to be built pretty strong to weather the storms in Nordhaus’ scenario: air temperature stabilises, but the ocean keeps heating up for quite a while.

  138. Ben McMillan says:

    In Nordhaus’ scenario, you get ~2 meters of sea level rise from the GIS. Also, in addition, you’ll get a few more meters from the Antarctic, glaciers, and warming oceans.

    I guess ‘cities below sea level, but surrounded by a massive wall to fend off the ocean’ is another amusing possibility. The ‘everywhere turns into the Netherlands’ option, but on steroids.

  139. bjchip says:

    The discount rate is negative !

    Nordhaus should stick to being an economist and stop pretending to be a scientist. Relying on Diaz for his damage estimate is just another piece of this.

    Assuming that that one source of sea level rise is the only thing going on at one time is the second card he’s palmed here.

    The sea level change by the end of the century can be up to 2 meters and is almost certainly more than 1 meter but “coastal adaptation” as Diaz assumes will be done to protect our communities is going to cease as it passes a half meter, because it will be quite obviously inadequate to what is coming. Yes, we can build something to protect against a 1 meter rise, even 1.5, but when we know that the rise can be more than 4 meters by 2200, why would we throw that money away? The only option then is retreat.

    https://iopscience.iop.org/article/10.1088/1748-9326/aacc76/pdf

    The next decade will give the lie to him in any case, as the CO2 from the late 1990’s starts to have its effect on our climate. The first danger is to our agriculture, from destabilization of the climate, the rainfall patterns and the droughts. If you don’t think that a stable climate is important, ask a farmer.

    Sea Level rise comes last, to flush the rats from the rubble of civilization.

  140. ecoquant says:

    Also, I find it odd that those arguing that we can’t extrapolate results of IAMs to the future will often point to these growth & climate studies and then make much larger extrapolations. Seems like a double standard to me.

    [1] IAMs don’t consider (a) limitations of our physiology, which are well known, and (b) structural breakage in ecosystems which happen too rapidly for selection processes to overcome, at least on the short term. In both instances, to the degree we rely upon having healthy bodies outdoors, in the first case, and to the degree we rely upon a plethora of ecosystem services, most of which have not been specifically categorized, we could be in trouble if tails on projections of temperature increases are realized. You act as if the mean of projections are the only story.

    [2] Holding the IPCC up as the extreme is laughable. The IPCC process had as its goal informing policy and policymakers, and, while it was made as insulated as possible from global politics and monied interests, just the big gap between the Advice to Policymakers — which is reviewed by member states — and the scientific consensus shows the pressures on the process. That process, too, overweights opinions of non-middle-of-the-road positions, particularly on the “it’ll all be okay” side. Why am I advocating asymmetry? Because there is way more room for probability support at the high ends of temperature than between the mean value and zero.

    [3] The IPCC determinations do not consider loss functions in their determinations. This is a casualty of trying to separate the science from the policy but it is a casualty nonetheless. A sensible approach is to do (modified) IPCC-like reviews but double-down on investment in geophysics and climate science to explore the many dark corners of the climate system, per, e.g., the recommendations on exploring possibilities of local bifurcations I’ve cited in other of my comments.

  141. ecoquant says:

    @Steven Mosher,

    Sea walls where it makes sense

    Sea walls almost never make sense.

    They are useless in Florida for geotechnical reasons. In other places they depend upon accurate projections of sea level rise over time, and also ignore that a bunch of flooding comes from the other direction …. From extreme precipitation which drains into harbors. Close the wall and you contain the flooding coming down rivers. This is what, for instance, is the fate of Providence, RI’s long-lived hurricane barrier.

  142. Ben McMillan says:

    Sarcasm: Oh, but you just carry the rivers to the sea in giant aqueducts. Problem solved!

  143. Mal Adapted says:

    Ben McMillan:

    From a sci-fi point of view, does that mean an array of deserted skyscapers sitting out in the ocean next to New-New-New-New York, or island-cities, far from the coast, built on the ruins/garbage of previous civilisations?

    Are you perhaps thinking of Kim Stanley Robinson’s 2017 novel New York 2140? A New Yorker review is here:

    This is the vision of the city in “New York 2140,” a science-fiction novel by Kim Stanley Robinson, out last month. It’s surprisingly utopian. New York may be underwater, but it’s better than ever. Sure, it was a rough hundred years for the planet: the seas rose ten feet in the two-thousand-fifties, then forty feet more around 2100, and billions of people died. Each episode of flooding was “a complete psychodrama decade, a meltdown in history, a breakdown in society, a refugee nightmare, an eco-catastrophe, the planet gone collectively nuts.” Now, though, New York is the “SuperVenice.”

    I submit that “science fiction” can be as useful as formal models at envisioning the future, and viewing the present from that perspective. The cost of climate change incurred b/w today and KSR’s world of 2140 is of little concern to its inhabitants, who seem pretty much like modern urban dwellers, though perhaps unrealistically civic-minded. Present day young people, OTOH, must dread the “complete psychodrama decades” ahead. I don’t expect to last beyond 2050 under optimal conditions, and I only hope what occurs before then is relatively mild. Meanwhile, I assuredly won’t be able to forget the billions of people (oh, the humanity) and millions of species (oh, the ecology) who will pay the highest price.

    OP:

    Globally, these economic analyses are reasonable, but they largely ignore the distribution of the impacts; losses could be huge in regions that are poor and don’t contribute much to global GDP.

    This is a primary source of tragedy in our 10-kyr drama of the planetary Commons. Not that I’d willingly disconnect from today’s global market, with all its long-standing externalities and injustices. Just that while I’m as narcissistic as anybody and more than some, thanks to science (including human behavioral sciences, h/t Biff Vernon) I can’t escape awareness of the tragedy!

  144. Mal Adapted says:

    Where, oh where, did my HTML tags go? Oh, well.

  145. Ben McMillan says:

    Mal Adapted: hadn’t heard of that KSR book. A decade after reading the Mars books I think I might be in the mood for another.

  146. Predicting the future is fun. The 2140 is 120 years from now. Today would be extraordinarily utopian to a person living 120 years ago in the year 1900:

    “Life expectancy for white Americans was just 48 years and just 33 years for African Americans–about the same as a peasant in early 19th century India. Today, Americans’ average life expectancy is 74 years for men and 79 for women. The gap in life expectancy between whites and non-whites has narrowed from 15 years to 7 years.

    In 1900, if a mother had four children, there was a fifty-fifty chance that one would die before the age of 5. At the same time, half of all young people lost a parent before they reached the age of 21.

    In 1900, the average family had an annual income of $3,000 (in today’s dollars). The family had no indoor plumbing, no phone, and no car. About half of all American children lived in poverty. Most teens did not attend school; instead, they labored in factories or fields.”

    http://www.digitalhistory.uh.edu/disp_textbook.cfm?smtID=2&psid=3175

  147. Ben McMillan says:

    Back on Nordhaus’ model, it is amusing that he chooses a ‘tipping point’ that takes ~10000 years to reach an equilibrium: as a consequence, glacial response is basically linear in the ~2000 year study. So why did they bother at all? It would be nice to have a result that doesn’t trivially follow from the assumptions.

  148. Sinocentric says:

    Mike said:

    “The world is not going to end due to a climate event, acute or otherwise. The sixth extinction, however, is going to increase in breadth, depth and severity and the potential for extinction of many, many species in the next 50 to 100 years is very high”

    No question about it. It’s a massive tragedy. Certainly there will be terrible human suffering as well. I also share your overall pessimism towards our getting the problem under reasonable control, it doesn’t seem likely right now. GDP is certainly not the only thing that matters in the world.

    As the blog author noted, there is somewhere between 2 degrees and 10 degrees where GDP will collapse. Maybe that’s where we find collective action, lol. Or maybe we will be more merciful to ourselves and other species and do something beforehand…I sure can’t say!

    -Kev

  149. mrkenfabian says:

    “The Drowned Cities” by Bacigalupi did it better than KSR in my opinion and cannot be accused of being utopian – militias stripping the salvage to buy guns and bullets to “eliminate the traitors” (every other militia) in an unending regional conflict. It took me a while to realise the (unnamed) setting was Washington DC and surrounds. I’m not sure the climate – or biological – science was accurate but it did capture the world changing seriousness; the portrayal of unreasoning and unconstrained human conflict as making things much, much worse – snatching defeat from the jaws of victory – seemed spot on.

    Not all the world is portrayed like that – some parts have adapted well. But it’s a world of haves and have-nots, with borders that are fiercely defended against (poor) refugees a feature.

  150. Steven Mosher says:

    nice article on SLR bj

    ‘We estimate a median global sea level rise up to 52 cm (25–87 cm, 5th–95th percentile) and up to
    63 cm (27−112 cm, 5th—95th percentile) for a temperature rise of 1.5 ◦C and 2.0 ◦C by 2100
    respectively. We also estimate global annual flood costs under these scenarios and find the difference
    of 11 cm global sea level rise in 2100 could result in additional losses of US$ 1.4 trillion per year
    (0.25% of global GDP) if no additional adaptation is assumed from the modelled adaptation in the
    base year. If warming is not kept to 2 ◦C, but follows a high emissions scenario (Representative
    Concentration Pathway 8.5), global annual flood costs without additional adaptation could increase
    to US$ 14 trillion per year and US$ 27 trillion per year for global sea level rise of 86 cm (median) and
    180 cm (95th percentile), reaching 2.8% of global GDP in 2100.”

    REPEAT AFTER ME… RCP 8.5 is NOT GUNNA HAPPEN.

  151. Willard says:

    > NOT GUNNA HAPPEN

    Yet our luckwarm fellowship does not profer such pronouncement regarding 1.5C.

    A sequel to the paper:

    As sea level is rising along many low-lying and densely populated coastal areas, affected communities are investing resources to assess and manage future socio-economic and ecological risks created by current and future sea level rise. Despite significant progress in the scientific understanding of the physical mechanisms contributing to sea level change, projections beyond 2050 remain highly uncertain. Here, we present recent developments in the probabilistic projections of coastal mean sea level rise by 2100, which provides a summary assessment of the relevant uncertainties. Probabilistic projections can be used directly in some of the decision frameworks adopted by coastal engineers for infrastructure design and land use planning. However, relying on a single probability distribution or a set of distributions based upon a common set of assumptions can understate true uncertainty and potentially misinform users. Here, we put the probabilistic projections published over the last 5 years into context.

    https://link.springer.com/article/10.1007/s10712-019-09550-y

    The limitations presented seem daunting.

  152. Willard says:

    > The discount rate is negative !

    Citation needed.

  153. Steven Mosher says:

    eco
    ‘Sea walls almost never make sense.”
    Gosh run and tell SFO that their barrier makes no sense because BOSTON! ur a civil
    engineering genius! who knew you could diagnose SFO defenses by looking at BOSTON!
    you should patent that shit!

    go tell these guys, they wasted their time. Tell them BUT BOSTON!

    https://www.climatecosts2040.org/

    https://www.climatecosts2040.org/download-the-report

    https://www.climatecosts2040.org/about-the-study

    To repeat. there are a couple options: Barriers, retreat.
    arguing that 1 of of these is much more likely than the other, doesnt change anything: two options.
    one, two. Not 1 ,2, 3; but rather 1 or 2 or 1 &2. You can STAY or you can GO. or do a little of both.

    Pro tip:
    The way you argue against an argument that there are two options is to demonstrate that there
    is only one, or more than 2. But I think it’s it obvious that if the water rises you have 2 options
    STAY or GO. maybe hover is a third, but thats just another way of staying.

    yes, some folks will find that barriers are too expensive
    https://www.theinvadingsea.com/2019/02/13/378-million-price-tag-to-prevent-flooding-leaves-delray-beach-reeling/
    They may have to GO. But that doesnt change the fact: two options; Stay or go.

  154. REPEAT AFTER ME… RCP 8.5 is NOT GUNNA HAPPEN

    Coming soon! From the authors of “Climategate: The CRUtape Letters”.

    “More riveting irrelevance updated to be irrelevant in 2019” – “Reality”

  155. ecoquant says:

    @Steven Mosher,

    No, dichotomies almost never make sense. There are not just two choices, and even in Boston and Cambridge, “go” is not one of them. The answer in both Cambridge and Boston is to “sort of stay”, but demand, on some quick but reasonable schedule, that (a) all resources and equipment of value are no longer stored in the basement, or on the first and even sometimes the second floors, and (b) that basement, first, and second floors can be periodically flooded by seawater and brackish water without harm. (MIT is a special case, as they already have much infrastructure located below the level of the Charles River. They’re dealing with it their own way.)

    So, for downtown, the plan is even more overt … The commercial space will eventually be given over to a series of floodable parks. It’s an interesting project, because some of the most suitable plantings for such spaces are considered invasives by the limitations of modern day normative Botany.

  156. Ben McMillan says:

    If RCP8.5 is not going to happen, then something is very wrong with Nordhaus’ model, because his ‘optimal’ pathway (i.e. with mitigation) isn’t that far from RCP8.5. In Nordhaus’ 1993 pathway you get to 3.5C by 2100 (good chance of much worse of course).

    So that really does look like pretty impressive sea level rise, especially in the longer term, even if the ice sheets don’t do anything surprising. Cities are building infrastructure now to protect them from sea level rise/storms, but not what would be needed in a world with 3m sea level rise.

    I think the main thing ‘wrong’ with Nordhaus’ model is that humans actually do care about the future more than a 4.5% discount rate suggests. We could just be burning more coal in the West, but aren’t, and because we’ve made a significant effort developing/deploying other tech, we have other cost-effective options. The ‘effort’ we’ve made so far is only a fraction of a percent of GDP, but has made a significant difference to likely trajectory.

    Also note that in Nordhaus 1993 you need a $300 per tonne carbon tax to get a 20% drop in emissions. Who thinks that is even vaguely correct?

  157. David B. Benson says:

    Of course, using new capture techniques, carbon dioxide can be removed from the atmosphere and permanently sequestered in basalt. There is vastly more than enough basalt:
    http://bravenewclimate.proboards.com/thread/522/carbon-sequestration-viable-solution-depends?page=1#post-6053

  158. Ben McMillan says:

    Sorry, that was potentially confusing: $300 per tonne carbon (~$80 per tonne CO2 in 1990 dollars, so ~$150 in current dollars) to reduce emissions in 2100 to 80% of 1990 levels.

  159. Nathan says:

    Wonder if they’ll modify their costs to the economy with this news…
    https://www.nature.com/articles/s41467-019-12874-3

    That’s not very good news…

  160. David B. Benson says:

    Nathan, why?

  161. Nathan says:

    David

    Their claim is that the end of the last interglacial had very rapid sea level rise:
    “Implied rates of sea-level rise are high (up to several meters per century; m c−1),”

    In this case a couple of periods of around 3m per century.
    “Finally, we infer intra-LIG sea-level rises with event-mean rates of rise of 2.8, 2.3, and 0.6 m c−1. Such high pre-anthropogenic values lend credibility to similar rates inferred from some ice-modelling approaches51. The apparent reality of such extreme pre-anthropogenic rates increases the likelihood of extreme sea-level rise in future centuries.”

  162. Greg Robie says:

    Piling on on NobelNordhaus, and from this paper on the Greenland Ice Sheet (https://www.pnas.org/content/116/25/12261 – assume, @-BenMc, this is what you referenced), an equivocation ad absurdum:

    “The present study may overestimate or underestimate the economic impacts of the GIS depending upon the nonlinear interactions between the GIS and [in context these “include the Antarctic ice sheet, Atlantic meridional overturning circulation changes, ocean carbonization, monsoons, tropical cyclones, and forests”, or], these other earth system changes. … then the impacts here [Nordhaus’ study] will be underestimates because the calculated marginal impacts are smaller than the true ones that include other impacts of warming. This shortcoming is an important qualification but must await studies of other systems to be evaluated.”

    I find it further disenginious that the study assumes the Greenland Ice Sheet was stable between 1961 & 1990. Inclusive of that period Arctic permafrost retreated 80 km north in Canada (based on indicators species and analysis of military arial photographs starting in the ’50s). The physics of the energy storage capacity of the latent heat of ice defines the asserted “stability” (and the resulting assumed temperature baseline that biases the subsequent calculations), approximates a geeky Saturday Night Live [science] skit. Isn’t the stated assumption regarding a baseline tantamount to being duplicitous … if motivated reasoning is not in play?

    sNAILmALEnotHAIL …but pace’n myself

    https://m.youtube.com/channel/UCeDkezgoyyZAlN7nW1tlfeA

    life is for learning

    so all my failures must mean

    that I’m wicked smart

    >

  163. Greg Robie says:

    @-DavidB, is it also correct that the basalt has to be at the right temperature, and at a technologically constrained ‘economical’ depth to mineralized an injected CO2 slurry? My understanding is that the Columbia Plateau and Iceland, where the concept has been piloted, qualify.

    …And then, isn’t there the pipeline and volume issues which constrain that linked assertion of abundance?

    sNAILmALEnotHAIL …but pace’n myself

    https://m.youtube.com/channel/UCeDkezgoyyZAlN7nW1tlfeA

    life is for learning

    so all my failures must mean

    that I’m wicked smart

    >

  164. Dave_Geologist says:

    David

    How long will it take and how much will it cost to extract 45 Mt of CO2 form the atmosphere. How big will the plant be and how much air needs to pass through it? Do you plan just to pass air through the basalt or will you extract the CO2 using, e.g., an amine reformer? AR technology generally operates on percent quantities of CO2. Is it commercially available for hundreds of ppm?

    How finely due you have to crush the basalt to allow the whole cubic kilometre to react? How long will that take, how much will it cost and where will you put it? Gravel-sized is about as small as you can go with a big pile. Powder will need special treatment and containment and probably stirring to keep the flow paths open and stop it clumping. Cost and time please. If gravel, what is the diffusion rate into each grain? How long will it take to saturate a grain? What percentage of the total volume can be treated as working volume in a decadal time frame of interest? 1%? then you need 100 cu km.

    I could go on, but my unicorn needs feeding.

  165. ecoquant says:

    @David B Benson, @Dave_Geologist,

    DaveG just wrote what I was going to say: Can they do it at scale? And have they priced it out? Is it lower than US$100/tonne CO2? Because if it isn’t much lower than that, the process will bankrupt even Nordhaus’ rosy economic picture: Dropping 70 ppm costs dozens of Gross World Products at US$100/tonne.

  166. ecoquant says:

    … to reduce emissions in 2100 to 80% of 1990 levels …

    And that will do what precisely? Atmospheric and climate CO2 concentrations will continue to go up. We need zero. And have we stopped emitting by then? Are you saying this is to offset our (roughly) 40 GtCO2 per annum?

  167. Ben McMillan says:

    @ecoquant: That is what Nordhaus considers a ‘drastic reduction in CO2’ but most climate scientists consider a completely inadequate target.

    Basically Nordhaus was (still is?) arguing that even very unambitious climate targets will be too hard.

    That’s what you get with a low damage function but discount rates and expense of mitigation turned up to 11.

  168. -1=e^iπ says:

    ATTP –

    “In terms of testing how society responds, I’m not sure this is true. Most people expect the impacts to grow non-linearly”

    1. There is an empirical literature that looks at the impact of climate on the economy (i.e. how the society responds), and IAMs use estimates from this literature in their damage functions.
    2. Damage functions in IAMs are non-linear and take non-linearities into account. Your should know this….

    “Yes, but some people argue that this isn’t a good empirical way of estimating how we will respond to similar levels of *global* warming.”

    If its the best way we have, then we should go with that. Just quantify your uncertainty properly, and then have some risk aversion in your integrated assessment model. Improve your estimates as you get better information, and then update the IAM to determine the new level of pigouvian taxation.

    “I’m pointing out that we’re extrapolating into a region of parameter space we’ve never experienced before.”

    By this logic, we shouldn’t believe the projections of general climate models. I’m surprised to hear you make an argument similar to what you might hear at a denier blog.

    Ben McMillan –

    “I guess ‘cities below sea level, but surrounded by a massive wall to fend off the ocean’ is another amusing possibility. The ‘everywhere turns into the Netherlands’ option, but on steroids.”

    See Diaz. https://link.springer.com/article/10.1007/s10584-016-1675-4

    bjchip –

    “The sea level change by the end of the century can be up to 2 meters and is almost certainly more than 1 meter but “coastal adaptation” as Diaz assumes will be done to protect our communities is going to cease as it passes a half meter”

    In Diaz, costs rise with sea level rise. I suggest you read the paper.

  169. -1,
    It’s almost like you’re not reading what I’m writing.

    1. There is an empirical literature that looks at the impact of climate on the economy (i.e. how the society responds), and IAMs use estimates from this literature in their damage functions.

    Yes, but it’s not obvious that the climate being used to determine these empirical estimates are directly comparable with the climate we’re going to experience (I don’t mean that we’ll necessarily see events we’ve never seen before, but it’s not obvious that changing the climate in one region means that the economy of that region will become like the economy that had a similar climate in the past).

    2. Damage functions in IAMs are non-linear and take non-linearities into account. Your should know this….

    Yes, but there are many non-linear function that may be similar for small changes, and very different for large changes. My point was that one could probably develop a different non-linear that satisfied the empirical relationships today, but behaved very different at higher levels of warming. How would we test this. I don’t think we can. As far as I can tell, we’re trusting that these damage functions will behave suitable out of sample (and this is still assuming that the sample is representative). They may not.

  170. David B. Benson says:

    To all & sundry — Form supercritical carbon dioxide. Pump hard to force it down a hole in a basalt formation. In a few years it has all reacted, exothermically mind you, so permanent.

    The details are found towards the end of the thread previously cited here, over on the BNC Discussion Forum site.

    I don’t know the costs, which depend upon how expensive electricity is. However, MIT engineers have invented a new method to capture carbon dioxide from the air which looks most promising. There is a link to an article about this, again placed on the BNC Discussion Forum Energy subsection.

  171. Ben McMillan says:

    -1=e^iπ:
    Well, I was discussing the long term scenario in the Nordhaus paper that you were praising where humans decide to try melting Greenland. I’m pointing out that this is a pretty silly misuse of an IAM, and outlines a nightmare scenario where we really would have to abandon cities or go “full Dutch”. Basically I’m trying to explain why everything will not be ‘just fine’ in that scenario.

    Diaz only discusses the short term (ie to 2100), and SLR of up to ~1m, where coastal adaption maybe looks feasible. The damage function would be significantly worse-than-quadratic for big sea level rise. You can get a feel for that by seeing how quickly the ‘retreat and protection’ costs rise at the end of the century.

  172. Dave_Geologist says:

    I note you didn’t mention costs, David. OK you’ll inject it into basalt, and 3% vesicular porosity sounds right. But you still have to extract the CO2. Why are we not doing that already? The technology has existed for decades to do that into sandstone reservoirs. The problem has not been a shortage of sandstone reservoirs. I agree that basalt is more attractive in some ways. But the costs would be the same as for sandstone, and the timeframe is up to ten times longer to fill the reservoir. Hmm, maybe there’s some other barrier than shortage of reservoirs?

    I’d be very wary of scaling up from a single core plug, especially when they’ve artificially fractured that plug. One fracture per inch is orders of magnitude smaller than is done in fraccing, and probably not technically possible in an open system because leakoff into early-formed fractures will ultimately prevent new ones forming. I found some old well tests for DC-6 which has a background permeability of 0.001mD and three short intervals of 0.1mD. So in the range of producing shale gas reservoirs, as you’d expect for a formation with almost zero matrix permeability, although towards the poorer end of the spectrum. The high-perm zones are disappointingly sparse, so to get CO2 away from the wellbore you’d need to generate lots of artificial fractures. Fair enough, but budget for a similar drilling and fraccing spend per sq km as you would in shale gas. You’ll need to budget more for refracs and workovers because the carbonate will clog up the fractures. Since it’s a conventional tight reservoir not a shale, you’d probably be better benchmarking it against something like the Khazzan gas field in Oman.

    Since unlike shales and sandstones, basalts have essentially no primary permeability between the pores which are vesicles, the 0.001mD likely represents background fractures. It could be weathered matrix, but from a practical viewpoint it doesn’t really matter. The high-permeability fractures (and 0.1mD, albeit averaged over a zone which includes matrix, is pretty low – we’d want 1-10mD for a naturally-producing fractured reservoir) are too far apart to contribute meaningfully. They’ll actually be a problem, acting as thief zones. Normally I’d want to know things like effective block size between the big natural fractures, and imbibition parameters, but this is like a shale: brute force, rather than nature, will provide. My working assumption would be that the actual pore volume accessed by the wells is comparable to the recovery factor in shale oil or gas: 3-6% of the hydrocarbons in place, 2-4% of the pore volume. So replace 1 cu km in your calculation with 30 cu km. I found a Barnett study recommending a 1400 ft well spacing, and let’s make them 1km long. So four wells per cu km, or 120 wells for your 45 Mt CO2. Let’s assume we’re emitting 45 Gt CO2 per year when we get it working. You need 120,000 wells. Baker Hughes has the global rig count at about 1200. Let’s say they each drill 10 wells a year. You need ten times the current global rig count. Disposal takes 40 years, so to maintain a steady state, you need a stock of about 5,000,000 wells, albeit drilled over 40 years. And more than 1,000,000 cu km of basalt to drill them into.

    It has attractions over sandstone disposal – the reservoir volumes are certainly there for example, and leakage risks should be low. But not zero: remember, rainwater dissolves calcium carbonate, albeit slowly, and the Colombia Plateau is a fractured, flowing aquifer. From my rough calculation, it looks similar to sandstone disposal. My reckoning there is that you’d need two or three times our current oil and gas producing infrastructure, because you’d also be covering for coal burning and cement production. At this level of analysis, two or three times or tens times are a wash. And neither is a magic bullet.

  173. ecoquant says:

    @Dave_Geoligist,

    Excellent stuff. I always learn tonnes when you write one of these technical comments. Love it.

    Thanks.

  174. bjchip says:

    I don’t pay for access to these things. Retired.

    I asserted and maintain that people aren’t going to pay for massive walls and protections that will not serve them for more than 50 years, and that they will not build extensive infrastructure behind protective structures that they know will fail them within 100. The assumptions of Diaz as inferred from the abstract, fail to account for human nature relating to risk and the effects of an increasing rate of sea level rise on human perceptions.

    Perhaps that meaning was not clear.

    I may be wrong too, Some people are known to live next to rivers and downstream from dams. I would not. Murphy is god.

    Nordhaus’s estimate of a damage function is risibly inadequate and his attempt to justify it using one of the slowest changing effects in climate is equally so.

  175. Dave_Geologist said:

    “Classical reservoir engineering can only handle single-porosity, single-permeability and dual-porosity, single-permeability models, not dual-porosity, dual-permeability models. In practice you can use SP-SP if the fracture permeability contributes less than 10% of the total, and DP-SP if the matrix permeability contributes less than 10% of the total. Otherwise you’re stuck, even with numerical simulators, for other than toy models. You don’t even have to go to discretisation: the equations explode and there is no analytical solution. “

    We solved that and have an analytical solution. The results are applied to characterize the depletion of fracked oil production.

  176. ecoquant says:

    This is an opportunity to drop in something I’ve wondered/thought about as it is pertinent to the discussion. I am strongly in favor of aggressive action on mitigating climate disruption, but I have, like many, been surprised/confused/made to wonder about the hesitancy of publics to take this on, and their representatives. I don’t buy that it’s entirely the fault of, saying “conniving fossil fuel majors”, although they have done their share. Indeed, despite their efforts it does look like we’re making some progress, even in the United States.

    Nevertheless classical Chicago School or Austrian Economics is not all there is. There are many instances of behavioral economics, and they have been rewarded in recent years by Nobel Prizes. There is the work of Kahneman and Twersky, for example. Kahneman’s pessimism on climate action has been noted. Kahneman, Smith, Thaler, Bannerjee, Duflo, and Kramer all won the Nobel with this kind of work.

    I recently listened to Dr Esther Duflo on Talking Politics (a great podcast, BTW), in part about her book with Dr Abhijit Bannerjee, on Good Economics for Hard Times. I will listen to it a second time.

    I was impressed with her discussion of “stickiness” in human economic behavior and how this goes against conventional economic wisdom and the policy based upon it, a discussion illustrated with many citations of cases and evidence. This is about the reluctance of people to pursue changes in their personal and familial practice even when there is evidence of substantial financial benefit to do so. It is most strongly illustrated there in the reluctance of people to move to different places for betterment, and informs a view of political economy where, for example, in the United States, despite the economic fortunes of many rural locales to decline (nominally thought about in the midwest but it applies to, for instance, upstate New York, too).

    So I wonder how much of the reluctance to get behind an aggressive program of mitigation is due to this kind of thing, as opposed to the Kahneman risk aversion.

  177. Willard says:

    > Damage functions in IAMs are non-linear

    Yet it took Burke, Hsiang, and Miguel to study the global non-linear effect of temperature on economic production. Vintage 2015. Their conclusion:

    We show that overall economic productivity is nonlinear in temperature for all countries, with productivity peaking at an annual average temperature of 13.6C and declining strongly at higher temperatures. The relationship is globally generalizable, unchanged since 1960, and apparent for agricultural and non-agricultural activity in both rich and poor countries. These results provide the first evidence that economic activity in all regions is coupled to the global climate and establish a new empirical foundation for modelling economic loss in response to climate change, with important implications.

    Click to access BurkeHsiangMiguel2015.pdf

    As Ted Miguel suggested in his World Bank Presentation, there are three myths that must die:

    1. That wealth insulates societies from the effects of climate;
    2. That economic productivity has become less sensitive to climate over time (adaptation);
    3. That agriculture is sensitive to climate but not other sectors.

    These three myths have been promoted in this thread by the usual suspects.

    I wonder why.

  178. Willard says:

    > I don’t pay for access to these things.

    ***

    I think I heard basalt:

  179. Dave_Geologist says:

    Willard, they’re turning a few percent of the basalt that is contacted by the injected fluid into altered basalt. Mostly carbonates and hydroxycarbonates. If they do it at high enough temperatures, probably some zeolites and stuff as well.

    Paul, dual-porosity, single-permeability should be more than adequate for shale gas. The length-scale difference is such that any microfractures will look like matrix permeability. Things like relative permeability will be wrong out of the box, but that’s what pseudos are for. If they’re close enough in scale to matter, so you need fully bidirectional flow, you tend to use a node-pipe-tank representation rather than a grid. And in that case I’ll take a lot of convincing that you can represent it analytically other than by curve-fitting (and if you can curve-fit it, classical reservoir engineering works anyway).

  180. Willard says:

    > they’re turning a few percent of the basalt that is contacted by the injected fluid into altered basalt. Mostly carbonates and hydroxycarbonates. If they do it at high enough temperatures, probably some zeolites and stuff as well.

    Sure, but is it any good?

  181. Dave_Geologist said:

    “The length-scale difference is such that any microfractures will look like matrix permeability. “

    Porosity has essentially the same behavioral effect as permeability, which explains why you’re making a mountain out of a molehill.

  182. David B. Benson says:

    Potentially a low cost way to remove carbon dioxide from air.

    Looks to me better than Lackner’s artificial trees.

    http://bravenewclimate.proboards.com/thread/522/carbon-sequestration-viable-solution-depends?page=1#post-6062

  183. Dave_Geologist says:

    Do the maths David. Low cost only if you mean in the order of several times the current spend and infrastructure footprint of oil and gas production and processing. Say 10-20% of global GDP to get to net zero on my rough calculation (O&G is 2-3% today, but that includes downstream stuff all the way to your car’s tank, which should be excluded). A bit less if you can get a better working volume, but that only scales the wells, not the capture, extraction and processing infrastructure. 5-10% would have been my estimate for sandstone reservoirs, but you’d probably run out of reservoirs and hit NIMBYism unless you put it all offshore, which would jack up costs. Small on the scale of fighting WWII, and about half the cost of Bloomberg’s 34% estimate of OAC’s Green New Deal, which I would presume is highballed. But that includes a bunch more stuff than climate action, like Medicaid for all, a living wage and better social security. So as a rough guide, BAU emissions but net zero by CO2 sequestration would cost about the same as the climate element of a full-fat GND. Obviously the industrial and social consequences of those two alternatives would be very different. I’ll believe BAU emitters are serious about sequestration when I hear them proposing that we drill 100k disposal wells per year and spend half what AOC wants to spend on the GND, but spend all of it on extraction and disposal technology.

    There are advantages and disadvantages of putting it in basalt vs. sandstone reservoirs. Maybe a bit more expensive, maybe a bit cheaper, different long-term behaviour (certainty of slow leakage vs. risk of sudden catastrophic leakage). We should probably do both. And anything else which minimises the cost and footprint by exploiting the local geology.

  184. Dave_Geologist says:

    No Paul, porosity does not have essentially the same behavioural effect as permeability, at least not outside of the realm of curve-fitting to very simple special cases. Which has a track record of failing out-of-sample tests.

    That’s why MSc students have to buy a 500-page book to learn about classical reservoir engineering. It’s more than decline curves in spreadsheets. Indeed that doesn’t even qualify, and spreadsheets weren’t around when it was developed in the 1950s. I recommend Laurie Dake’s classic, Fundamentals of Reservoir Engineering. Amazon have a very good deal on the paperback and Kindle versions.

    But let’s not get into another bun-fight. The key takeaway for me from my venture into the world of basalt is that we should think of basalt sequestration as operationally equivalent to shale gas, and sandstone sequestration as operationally equivalent to conventional oil and gas. With the same implications for people who pile into it, thinking it’s a magic bullet, and find to their regret that it’s not as easy or cheap as they thought it was.

  185. Dave_Geologist says:

    Just for fun, another rough calculation. When the UK Chancellor costed getting to net zero by 2050, the press made much of the scary-huge total number. Of course that totalised spend over 30 years, and was in money-of-the-day so included inflation. Independent analysts costed it at 1-2% of GDP per annum, assuming we start today. If we did it all at once, 30-60% of GDP. but in practice more, because it would be inefficient to scrap our transport fleet all at once, build 100 prototypes rather than a few and fed the rest in later when they’re cheaper, etc. So bigger than the sequestration numbers I estimate above, but the difference is that a lot of it is one-off investment, whereas sequestration involves ongoing costs. To do it by 2030, like XR want, would be 3-6%, but let’s call it 5-10% because you’ll be doing things like scrapping assets half-way through their useful life. We’d certainly notice that. 1-2% would probably be noticeable. It’s on a par with the post-2008 fall in growth rate, so if we get back to normal growth rates, every decade would feel like the last decade. We’d have visible infrastructure changes which some would consider a price worth paying, but the main benefit would be climate change damages foregone, which would be intangible to most people.

    Deciding the best mix of responses is the sort of thing economic models should be used for, although perhaps not DICE-type models. Much easier to cost a well or an electric car than to cost 50cm of SLR or 20k people dead in a heatwave. Rather than drilling 100k disposal wells per year, is it better to drill fewer wells while the same time replacing FF power stations with nukes and/or renewables? How does the cost of early scrappage and replacement of FF vehicles compare to the cost of CO2 disposal? Can you run the vehicles to end-of-life and amortise the disposal wells by using them to sequester FF emissions for the first decade, and harder-to-eliminate cement emissions in subsequent decades? I won’t bother doing that optimisation because I bet it’s been done already, and that’s what the Chancellor’s numbers were based on: the optimised 30-year programme.

  186. Steven Mosher says:

    willard, glad you enjoy peter. we shared some time at NU, he was leaving I was coming

  187. Greg Robie says:

    @-izen, & re tri-part polarization from the depolarization thread, with Nordhaus using a baseline temperature that provides a .3°C leg up regarding economic consequences, and using such to subsequently suggest that the resulting [rosey] GIS calculations ALSO have agency regarding other delayed response systems in the climate system, this “third” part is simply THE part the motivated reasoning, for a season, ‘disappeared’.

    For me, “…and Then There’s Physics” translates, roughly, there is one part. THEpart is leaking out all around the edges of a polarized, but complimentary, falsely-dichotomized-into-parts systemic motivated reasoning.

    @-eco’s Nobel-winning Duflo interview – linked above – included her sharing that macro-economic dynamics are “above her pay grade”. *SIGH*! But @eco’s assertion in the depolarize thread regarding choice and markets makes sense only to the degree the markets are not rigged on the macro level regarding reality/physics.

    Limited liability law enabled markets, and the motivated reasoning, which such a responsibility-free social construct enables, in turn engenders/informs that motivated reasoning. This effects a positive feedback cycle. In the US were are around 10 generations into this cycle and its psychological and sociological consequences. I think the UK is a bit further down this rabbit hole, and Royally so/more insidiously so.

    Therefore, and in terms of stories that can increase the possibility of social non-violence within the unfolding ‘chaos’, is this a possibility? With the ‘chaos’ being a consequence of – even feature of – a trusted motivated reasoning regarding CapitalismFail, is the ‘micro-story’ concerning the two constants in life being death and taxes one which can be expanded to be tri-part and include collapse; redefine collapse as unsurprising?

    sNAILmALEnotHAIL …but pace’n myself

    https://m.youtube.com/channel/UCeDkezgoyyZAlN7nW1tlfeA

    life is for learning

    so all my failures must mean

    that I’m wicked smart

    >

  188. David B. Benson says:
  189. Uli says:

    I would think to consider the following:

    In a far richer world a 10% damage would also mean a far higher absolute damage. Why should we accept this high absolute damage?

    The damage will be unequal to the sectors. F.e. in the US today only 1% of the GDP is due the agricultural sector. Reducing the agricultural output by 50% or more will directly effect the GDP only marginal, far less the 10%. If this happens only locally food could be imported.
    If this happens globally, we have a problem even in a very rich world with a damage of far less than 10% of the GDP.
    That automatic industries that mass producing electronic devices and consumer goods (other than food) been not reduced does not help much, but obscures the real problem by decreasing the damage relatively to the GDP.

    The next point is that this large absolute damage can be avoided at almost no cost.

    Lets calculate:

    The current world primary energy consumption is about 600 Exajoules per year.

    If the world population rise to above 10 billion and stabilizes there and the primary energy consumption per capita increases to that of the rich countries today, the world primary energy consumption would rise to about 3000 Exajoules per year.

    In about 33000 years, maybe the timescale of economic impact of climate change, the total primary energy consumption would be 100 Million Exajoules.

    For comparison, the fossil reserves are about 40000 Exajoules. If including the fossil resources it would be 580000 Exajoules. Therefore far more than 99.9% (or 99%, in case of resources) of the future primary energy consumption will come from non-fossil energy anyway.

    Would it be economic worthwhile to take a loss of 10% of the GDP, just for generating only 0.04% (or 0.58% in the case of resources) of the global primary energy by fossil fuels?

    Assuming only 1000 trillion dollar (*) GDP in this richer world and a 10% damage from burning the fossil reserves only, this translates to a damage of about half a million dollar per barrel of fossil oil.

    Therefore the damage will rise in a far richer world to very high and unacceptable absolute values.

    Generating the same amount of energy from non-fossil fuels without this damage will be and is even today far far cheaper.

    (*) in today value. It may be higher, if the energy efficiency $GDP/Joule is high.

  190. Pingback: 2019: A year in review | …and Then There's Physics

Leave a comment

This site uses Akismet to reduce spam. Learn how your comment data is processed.